Naked Science Forum

Non Life Sciences => Physics, Astronomy & Cosmology => Topic started by: amrit on 19/05/2010 08:27:53

Title: Will a photon clock run at a different rate from an atomic clock under gravity?
Post by: amrit on 19/05/2010 08:27:53
We have a “photon clock” made out of two mirrors A and B. Photon is moving from A to B, back to A and so on. One traveling of the photon between A and B is a “tick” of the clock. We take two photon clocks. One photon clock is on the surface of the earth, second is 4200 meters below at the bottom of the mine shaft. Velocity of light is invariant on gravity; both of clocks will “tick” with the same velocity.

We take two atomic clocks. One clock we put beside photon clock on the surface and second beside clock that is 4200 meters deep. According to the relativistic gravitational effect of relative velocity of material change second atom clock will in 30 days “tick” faster as the atom clock on the surface for 1,23 x 10 -6 seconds.

General Theory of Relativity considers light moves through the space with constant velocity regardless upon the strength of gravitation. This implies that at the scale of the photon and below at the scale of Planck relativistic gravitational effect of relative velocity of material change does not exist.

yours amrit

Amrit, I've rephrased your title as a question to make the forum easier to navigate.  Try to do so in the future.  Thanks.  -Mod
Title: Re: Will a photon clock run at a different rate from an atomic clock under gravity?
Post by: Soul Surfer on 19/05/2010 09:16:24
You are thinking wrongly here.  Time always ticks at the same rate wherever you are and however you move,  so you would not notice any change.  The difference in time scales is between the two points one inside and one outside the gravitational field and/or  travelling at different speeds. so it is the distant observer who will see the clocks ticking at a different rate. and you will only notice the difference when you get back and compare your clock with that of the distant observer.
Title: Re: Will a photon clock run at a different rate from an atomic clock under gravity?
Post by: amrit on 19/05/2010 12:57:36
I would not agree.
Atomic clock on the top of a high mountain runs faster than at the sea-side.
This is proved with experiment.
We will prove with experiment that for the photon clocks it is not so.
They run with the same velocity regardless to the gravity.

amrit
Title: Re: Will a photon clock run at a different rate from an atomic clock under gravity?
Post by: graham.d on 19/05/2010 13:48:17
At first sight your reasoning seems sound, Amrit. It is an extension to the simple demonstration of time dilation in Special Relativity where a stationary observer is watching light bouncing between two mirrors in a moving frame. This is interesting.

Have you any theory associated with this?
Title: Re: Will a photon clock run at a different rate from an atomic clock under gravity?
Post by: syhprum on 19/05/2010 13:58:16
"General Theory of Relativity considers light moves through the space with constant velocity regardless upon the strength of gravitation"
This is true of course but when there is a gravitational field present the light takes a curved path hence the effective speed is less than if it took the direct path.
Title: Re: Will a photon clock run at a different rate from an atomic clock under gravity?
Post by: imatfaal on 19/05/2010 14:46:24
Amrit - a bit confused.  You are certainly right about the atomic clocks running slower due to a time dilation effect caused by gravity - but why would this time dilation effect not alter the perceived time for photon ticks as well. the speed of light will remain constant for a local observer - ie both clocks must remain in synchrony.  on the surface an observer will notice that time is slower in the mine compared to his measurement
Title: Re: Will a photon clock run at a different rate from an atomic clock under gravity?
Post by: graham.d on 19/05/2010 15:04:54
I think that the photon clock will behave the same as the atomic clock though I do not agree with Syphrum. The maths would not work out - the bending of light would not increase the path length sufficiently. It is not the gravitational field that causes the time dilation but the gravitational potential. If you have a clock at the centre of the earth there is no significant field but the clock will be seen to run slower that one (also in no significant field) close to an observer a long way from the earth. The clock at the centre of the earth is at a much lower potential; another way to look at it is that the ticks from the clock would be redshifted in emerging from the earth's gravity to reach the distant observer.

I don't think a photon clock will behave differently but I have not reasoned out why yet. I think it is to do with not being able to time photon speed remotely. It can only be done by observing events and their intervals.
Title: Re: Will a photon clock run at a different rate from an atomic clock under gravity?
Post by: JP on 19/05/2010 15:50:32
I don't think a photon clock will behave differently but I have not reasoned out why yet. I think it is to do with not being able to time photon speed remotely. It can only be done by observing events and their intervals.

I think so.  It seems to me that to compare the clocks you have to get a signal from one clock to the other.  In doing so, the signal itself is going to be distorted because it has to change from the center-of-earth coordinate frame to the far-from-earth coordinate frame, which would make the results not agree.  It would be as if you took two sheets of graph paper with different scales and gave them to two of your friends.  You ask each of them to draw a line that's 5 boxes long.  Each friend would tell you they've drawn a 5-box-long line, and only when you compare them would you be able to tell the difference.  General relativity basically tells you that your clocks are going to measure things on your local graph paper.  You have to compare them with someone else's graph paper to notice where they disagree.

Don't ask me to do the math though.... [???]
Title: Re: Will a photon clock run at a different rate from an atomic clock under gravity?
Post by: amrit on 19/05/2010 19:19:49
At first sight your reasoning seems sound, Amrit. It is an extension to the simple demonstration of time dilation in Special Relativity where a stationary observer is watching light bouncing between two mirrors in a moving frame. This is interesting.

Have you any theory associated with this?

Yes I have, see our last article published in Physics Essays:
http://scitation.aip.org/getabs/servlet/GetabsServlet?prog=normal&id=PHESEM000023000002000330000001&idtype=cvips&gifs=yes&ref=no

yors amrit
Title: Re: Will a photon clock run at a different rate from an atomic clock under gravity?
Post by: amrit on 19/05/2010 19:24:40
Amrit - a bit confused.  You are certainly right about the atomic clocks running slower due to a time dilation effect caused by gravity - but why would this time dilation effect not alter the perceived time for photon ticks as well. the speed of light will remain constant for a local observer - ie both clocks must remain in synchrony.  on the surface an observer will notice that time is slower in the mine compared to his measurement

Velocity of photon is invariant on gravity. Out of that comes that photon clock will not have relativistic gravitational effect. I got that insight dhuring 4 days of za-zen meditation.
CALTECH has such a clock. I plan to have 4 clocks, 2 atom 2 photon and do experiment next year in South Africa in Gloden Mine Shaft 4200 m deep.
If someone is interested he can join us

HERE IS PAGE OF CALTECH PHOTON CLOCK
http://www.physorg.com/news5577.html
Title: Re: Will a photon clock run at a different rate from an atomic clock under gravity?
Post by: imatfaal on 19/05/2010 22:02:29

Velocity of photon is invariant on gravity. Out of that comes that photon clock will not have relativistic gravitational effect.

But the slowing of the photon clock is not premised on the concept that the speed of light has changed - quite the opposite, to the observer it is invariable.  it is time that is dilated - the passing of time has slowed.  If i have understood correctly it is not that the measuring-devices that display the passage of time are incorrect; but that the passing of time has slowed ie proper time is still displayed.

how big and expensive are these clocks?  I would have thought very...

Gonna go to caltech site now. thnks for link

Matthew
Title: Re: Will a photon clock run at a different rate from an atomic clock under gravity?
Post by: imatfaal on 19/05/2010 22:20:17

HERE IS PAGE OF CALTECH PHOTON CLOCK
http://www.physorg.com/news5577.html

Amrit - Sorry to write consecutive posts but I think you might have put in a dud link. 

The link you gave and the paper that the physorg paper summarises is not for a photon clock that depends of the passage of light between two mirrors!!  It is a micromechanical device that is forced into oscillation by radiation-pressure; it is an analogue of a quartz clock that is forced into oscillation by electric current but using radiation-pressure.

I must admit that I thought and still kinda think that the photon clock based on a beam of light between two mirrors was an elegant gedanken to help elucidate SR.

Matthew
Title: Will a photon clock run at a different rate from an atomic clock under gravity?
Post by: amrit on 20/05/2010 14:56:07
Matthew

you think it is possible to make a real Photon Clock with two mirrors ?
If yes, than we will do an interesting experiment.

yours amrit
Title: Will a photon clock run at a different rate from an atomic clock under gravity?
Post by: imatfaal on 20/05/2010 15:58:07
Matthew

you think it is possible to make a real Photon Clock with two mirrors ?
If yes, than we will do an interesting experiment.

yours amrit

Amrit - No I don't think it is possible , I think it is a Gedankenexperiment; ie a thought experiment to help people to get their heads around SR.  On top of that I think your concept that photon clock (if it existed) would be unaffected by gravitational potential time dilation is incorrect.  I must admit I have grave doubts about your initial statements; they challenge very profound and important ideas - and I do not think you have backed them up enough yet.

Matthew
Title: Will a photon clock run at a different rate from an atomic clock under gravity?
Post by: amrit on 21/05/2010 00:34:07
Mathew is interesting link where Einstein discuss the subject:
http://www.relativitybook.com/resources/Einstein_gravity.html
important to know he never talks about fourth coordinate X4 as time,
he presented as X4 = ict.

A ) if light velocity is constant than photon clock "ticks" with equal velocity regardless gravity
B ) if light velocity is influenced by gravity change than photon clock "ticks" with different velocity in different gravity
What you think is real ?

We all agree that c = frequency •  vawe lenght
by gravitational red shift frequency goes down and vawe lenght goes up. For me this shows gravity does not influences velocity of light, A is real B is wrong.

yours amrit
Title: Will a photon clock run at a different rate from an atomic clock under gravity?
Post by: JP on 21/05/2010 01:48:54
He did that to make all the coordinates have the same units so that you can use (x1,x2,x3,x4) as a four-vector...
Title: Will a photon clock run at a different rate from an atomic clock under gravity?
Post by: amrit on 21/05/2010 08:07:13
SR is postulated on constancy of light velocity.
Equality of inertial and gravitational mass is connecting SR and GR.
Discussing on possibility that gravity infects velocity of light put under question relatedness of SR and GR.
Gravitational red shift shows that gravity influences only frequency and not velocity of light.
This means that “thought experiment” with photonic clock is correct:
Velocity of photonic clock is invariant on gravity.
In stronger gravity photon moving between mirror A and B change only frequency and not velocity.
And this means that relativity gravitational effect of relative velocity of material change starts above photon scale.
Title: Will a photon clock run at a different rate from an atomic clock under gravity?
Post by: JP on 21/05/2010 08:51:19
Gravity effects space and time.  This is why it can cause disagreement between your photon clocks at the center of the earth and in deep space.  Gravity doesn't have to effect the velocity of light in order to cause the clocks to disagree, since it effects the spatial distance between them.  Are you arguing that the clocks at the center of the earth and in deep space will agree perfectly when you compare them?
Title: Will a photon clock run at a different rate from an atomic clock under gravity?
Post by: graham.d on 21/05/2010 08:55:18
A proper treatment should use GR but here is my reasoning; as I said before it is importent to understand what is meant by measuring times and lengths - you should really measure events and intervals as these would be invariant. Please bear with this lengthy description.

I prefer to use two observers - one at a long distance from the earth and another at the centre of the earth (ignoring practical difficulties that do not change the gedanken experiment). The reason is that neither will be in a gravitational field (or at least not one that cannot be made arbitrarily small) but the one at the earth's centre is at a lower gravitational potential.

The photon clock is constructed of two miirors spaced apart by a distance "x" so that a "tick" would be 2x/c long. In taking such a clock to the centre of the earth you, as a local observer, would not see any change in its behaviour. The mirrors will be the same distance apart however measured with any method you would have locally and the ticks would appear at the same time interval as when you constructed it (using measuring sticks, other clocks etc).

To a distant observer though, you and your clock have moved to a lower gravitational potential. The ticks from your clock (and any other clock) will be redshifted and running slow compared with his clocks. The apparent conundrum is why, if the velocity of light is the same to all observers, should the ticks from the photon clock be slow? The answer is (I think) that, at the lower potential, lengths (as measured by the distant observer) are larger so the mirrors are further apart. It is importent to understand how a distant observer would measure lengths and this is fundamental in solving even some apparent paradoxes in Special Relativity. The distant observer could measure the mirror spacing by sending a light pulse that partially reflects of the back of the first mirror (closest to him) but also passes by and reflects of the front face of the second mirror (furthest away from him). He would receive, some time later, two pulses with a time interval between them. This time interval (at the centre of the earth) would be the same as the tick time, but it would also be redshifted in returning to the distant observer. Like the tick, he would see that the gravitational potential gradient would have stretched the measured time to 2x'/c. This is interpreted exactly in the way that Lorentz contraction in SR is defined. It is the only way a distant observer can measure distances and lengths. The mirror spacing is larger to the distant observer.

The correct and more rigorous treatment would use GR and intervals in space-time but is mathematically challenging - at least for me.
Title: Will a photon clock run at a different rate from an atomic clock under gravity?
Post by: imatfaal on 21/05/2010 10:46:30
Amrit - I realise from your paper and other points you have made that you are trying to investigate new concepts of time - but in order to do this you are advancing arguments and quoting sources that explicitly require an engagement with the effet of gravity on space and time; and you seem to be avoiding this.  Your photon clocks will disagree for reasons that JP and Graham mentioned above.

With regards to your link of Einstein's notes - please read the final few paragraphs of section 3 and advise how this does not compromise your entire argument.  you will note that in the argument Einstein does not define 4th dimnsion attributes at all but talks of three-dimensional frames of reference in either uniform acceleration or homogeneous gravitational field.

in order to prove your points you must deal with observed and theoretical predicted time dilation and with spacial distortion (as described very clearly by Graham above) - which you have not done so far.

Matthew
Title: Will a photon clock run at a different rate from an atomic clock under gravity?
Post by: amrit on 21/05/2010 13:15:32
Amrit - I realise from your paper and other points you have made that you are trying to investigate new concepts of time - but in order to do this you are advancing arguments and quoting sources that explicitly require an engagement with the effet of gravity on space and time; and you seem to be avoiding this.  Your photon clocks will disagree for reasons that JP and Graham mentioned above.

With regards to your link of Einstein's notes - please read the final few paragraphs of section 3 and advise how this does not compromise your entire argument.  you will note that in the argument Einstein does not define 4th dimnsion attributes at all but talks of three-dimensional frames of reference in either uniform acceleration or homogeneous gravitational field.

in order to prove your points you must deal with observed and theoretical predicted time dilation and with spacial distortion (as described very clearly by Graham above) - which you have not done so far.

Matthew

Matthew space is timeless.
With clocks we measure numeric order of material change i.e. motion running in space.
Fundamental unit of numeric order is Planck Time.
Velocity of atomic clocks depends on gravity.
Velocity of photon clocks is independent on gravity as C is constant.
This has nothing to do with close or distant observer.
Such are rules of the universe, we observe it or not.
http://www.vetrnica.net/index.php?option=com_content&view=article&id=21&Itemid=23
yours amrit
Title: Will a photon clock run at a different rate from an atomic clock under gravity?
Post by: amrit on 21/05/2010 13:22:01
Gravity effects space and time.  This is why it can cause disagreement between your photon clocks at the center of the earth and in deep space.  Gravity doesn't have to effect the velocity of light in order to cause the clocks to disagree, since it effects the spatial distance between them.  Are you arguing that the clocks at the center of the earth and in deep space will agree perfectly when you compare them?

as C is invariant on inertial system and gravity, velocity of photon clocks is the same in the whole universe
velocity of atom clocks is not the same.....we know why and when
Title: Will a photon clock run at a different rate from an atomic clock under gravity?
Post by: graham.d on 21/05/2010 13:33:04
Amrit, (to quote from Monty Python) this is contradiction, not argument.

"Space is timeless" is meaningless unless you explain your definitions
"'Velocity' of clocks" is also not what you mean (I think).
It has everything to do with the observer and the different gravitational potential. If you were to do the maths rigorously you would find the "spacetime interval" will be the same to all observers.
The rules of the universe are what they are and not what you choose them to be, so by all means do your experiment, but you seem to have presupposed the result. But if it turned out you were right you will surprise a lot of people :-)
Title: Will a photon clock run at a different rate from an atomic clock under gravity?
Post by: imatfaal on 21/05/2010 13:58:44
Amrit - I am afraid you are repeating assertions without providing anything more.  Would you care to explain how space can be timeless yet there is experimental and practical proof of time dilation through differing gravitational potential and relative velocity.  you keep on repeating that photon clocks are unvarying despite Graham's explanation under SR of why they vary - could you answer this single question.  

As an aside - I read your paper and flicked through the references; I was unable to find the quote you gave within the Eckle paper.  If, in fact, it was a paraphrase it should really not be in quotes - perhaps you could direct me towards it.
Matthew
 
Title: Will a photon clock run at a different rate from an atomic clock under gravity?
Post by: Farsight on 21/05/2010 15:29:02
We have a “photon clock” made out of two mirrors A and B. Photon is moving from A to B, back to A and so on. One traveling of the photon between A and B is a “tick” of the clock. We take two photon clocks. One photon clock is on the surface of the earth, second is 4200 meters below at the bottom of the mine shaft. Velocity of light is invariant on gravity; both of clocks will “tick” with the same velocity.
Hi amrit. I'm afraid this isn't right. The second clock runs slower than the first clock. People say the speed of light doesn't change, but it does. The reason why is simple: speed is distance over time. If we avoid radial length contraction by lying our clocks flat, the distance is the same for both clocks. However we say that time dilation has occurred for the second clock, and that the times are not the same. Speed equals distance over time, so if the distances are the same and the times aren't, the speeds aren't the same either, even though we measure them both to be 299,792,458 metres per second. There's a hidden scale-change at work here, wherein the clock goes slower not because "time goes slower", but because the light goes slower.
Title: Will a photon clock run at a different rate from an atomic clock under gravity?
Post by: graham.d on 21/05/2010 15:47:47
Can you show evidence for this slow light, Farsight. I agree with your conclusion but is it not spacial changes with gravitational potential rather than lightspeed changes? Isn't what you are saying a more classical view than a relativistic one? The view of a blackhole, from this perspective, is that light does not emerge because it can't reach escape velocity rather than it being redshifted to zero energy. Perhaps the two views are indistinguishable. Gravitational redshift is not normally thought of to result from slow light - though I seem to remember some old theory about this.
Title: Will a photon clock run at a different rate from an atomic clock under gravity?
Post by: amrit on 21/05/2010 19:43:34
Amrit - I am afraid you are repeating assertions without providing anything more.  Would you care to explain how space can be timeless yet there is experimental and practical proof of time dilation through differing gravitational potential and relative velocity.  you keep on repeating that photon clocks are unvarying despite Graham's explanation under SR of why they vary - could you answer this single question.  

As an aside - I read your paper and flicked through the references; I was unable to find the quote you gave within the Eckle paper.  If, in fact, it was a paraphrase it should really not be in quotes - perhaps you could direct me towards it.
Matthew
 

Matthew there is no a single experimental data that time exist. With clocks we measure numerical order of events running in space. Gravity influences atom clocks and do not influences photon clocks. This is my answer to you.
About Eckle you find his papers on arxiv.

yours amrit
Title: Will a photon clock run at a different rate from an atomic clock under gravity?
Post by: amrit on 21/05/2010 19:46:35
Amrit, (to quote from Monty Python) this is contradiction, not argument.

"Space is timeless" is meaningless unless you explain your definitions
"'Velocity' of clocks" is also not what you mean (I think).
It has everything to do with the observer and the different gravitational potential. If you were to do the maths rigorously you would find the "spacetime interval" will be the same to all observers.
The rules of the universe are what they are and not what you choose them to be, so by all means do your experiment, but you seem to have presupposed the result. But if it turned out you were right you will surprise a lot of people :-)

Space-time is timeless by the very definition of it. X4 is not time dimension, X4 is spatial too
X4 = ict ( as d = v x t)
Title: Will a photon clock run at a different rate from an atomic clock under gravity?
Post by: amrit on 21/05/2010 19:49:06
We have a “photon clock” made out of two mirrors A and B. Photon is moving from A to B, back to A and so on. One traveling of the photon between A and B is a “tick” of the clock. We take two photon clocks. One photon clock is on the surface of the earth, second is 4200 meters below at the bottom of the mine shaft. Velocity of light is invariant on gravity; both of clocks will “tick” with the same velocity.
Hi amrit. I'm afraid this isn't right. The second clock runs slower than the first clock. People say the speed of light doesn't change, but it does. The reason why is simple: speed is distance over time. If we avoid radial length contraction by lying our clocks flat, the distance is the same for both clocks. However we say that time dilation has occurred for the second clock, and that the times are not the same. Speed equals distance over time, so if the distances are the same and the times aren't, the speeds aren't the same either, even though we measure them both to be 299,792,458 metres per second. There's a hidden scale-change at work here, wherein the clock goes slower not because "time goes slower", but because the light goes slower.

speed is distance over time...........No it is not
v = d/tn where tn is numerical order of event that we measure with clock
Title: Will a photon clock run at a different rate from an atomic clock under gravity?
Post by: imatfaal on 21/05/2010 21:09:58
Armit

One a technical note - I have read the entire paper you referenced and could not find your quote.  Maybe I have missed it - but most academics take citations very seriously and it is important to get this simple thing right. 

I think I will leave this discussion at this point - the existence of time is so central to physics it requires more than assertion to shake my belief in it.  The mathematics and theory behind special and general relativity have been tested (including with regard to time dilation) probably more than any other theory in history. 

Good luck with developing your theories - please bear in mind my comments regarding getting your citations and references perfect.

Matthew 
Title: Will a photon clock run at a different rate from an atomic clock under gravity?
Post by: syhprum on 21/05/2010 21:22:42
What is the difference between a 'Photon clock' and the resonant cavity with which we are all familiar ?.
Title: Will a photon clock run at a different rate from an atomic clock under gravity?
Post by: Farsight on 22/05/2010 09:19:34
Can you show evidence for this slow light, Farsight. I agree with your conclusion but is it not spacial changes with gravitational potential rather than lightspeed changes?
The evidence is the different readings in the light clocks. In the second clock that was down the mine, the light has bounced back and forth fewer times than in the first clock. The spatial change is radial length contraction, so when you hold your clocks flat you avoid this. 

Isn't what you are saying a more classical view than a relativistic one?
No. The distinction is between Einstein's original general relativity and what's called the modern interpretation. See this article on the Baez website called is the speed of light constant? (http://www.xs4all.nl/~johanw/PhysFAQ/Relativity/SpeedOfLight/speed_of_light.html)

Einstein went on to discover a more general theory of relativity which explained gravity in terms of curved spacetime, and he talked about the speed of light changing in this new theory.  In the 1920 book "Relativity: the special and general theory" he wrote: . . . "according to the general theory of relativity, the law of the constancy of the velocity of light in vacuo, which constitutes one of the two fundamental assumptions in the special theory of relativity [. . .] cannot claim any unlimited validity.  A curvature of rays of light can only take place when the velocity of propagation of light varies with position." Since Einstein talks of velocity (a vector quantity: speed with direction) rather than speed alone, it is not clear that he meant the speed will change, but the reference to special relativity suggests that he did mean so.  This interpretation is perfectly valid and makes good physical sense, but a more modern interpretation is that the speed of light is constant in general relativity.

The view of a blackhole, from this perspective, is that light does not emerge because it can't reach escape velocity rather than it being redshifted to zero energy. Perhaps the two views are indistinguishable. Gravitational redshift is not normally thought of to result from slow light - though I seem to remember some old theory about this.
It isn't quite like that graham. You just switch the emphasis to the coordinate speed of light, and downgrade the locally-measured 299,792,458 m/s. At the black hole event horizon the time dilation is infinite and the coordinate speed of light is zero. Hence the light doesn't emerge because it's effectively stopped. This is what's called the "Weinberg field interpretation" as opposed to the "Misner/Thorne/Wheeler" geometrical interpretation of general relativity.
Title: Will a photon clock run at a different rate from an atomic clock under gravity?
Post by: Farsight on 22/05/2010 09:40:34
Amrit: atomic clocks are light clocks! See the NIST caesium fountain clock (http://tf.nist.gov/cesium/fountain.htm), and see http://en.wikipedia.org/wiki/Second (http://en.wikipedia.org/wiki/Second) for the definition of the second:

"Since 1967, the second has been defined to be the duration of 9,192,631,770 periods of the radiation corresponding to the transition between the two hyperfine levels of the ground state of the caesium 133 atom. This definition refers to a caesium atom at rest at a temperature of 0 K (absolute zero), and with appropriate corrections for gravitational time dilation."

In the NIST fountain clock, lasers and a microwave cavity are employed to cause hyperfine transitions, which are electron "spin-flips" within caesium atoms. These are electromagnetic events that emit microwaves, light in the wider sense, which is received by a detector. The detector counts incoming microwave peaks. When it gets to 9,192,631,770 we say a second has elapsed. The frequency of the light is then 9,192,631,770 Hz by definition.

Note the mention of gravitational time dilation above. If you were to take this clock and place it in a region of low gravitational potential, it would be like pressing a slow-motion button. All electromagnetic and other processes would then occur at a reduced rate, including the hyperfine transition and the motion of the light towards the detector. Again when the detectors gets to 9,192,631,770, we say a second has elapsed. But it's important to realise here that in this situation, the light is moving slower and this is why the second is bigger. We then use this second... to measure the speed of light. That's why we always measure the local speed of light in vacuo to be 299,792,458 m/s.
Title: Will a photon clock run at a different rate from an atomic clock under gravity?
Post by: Geezer on 22/05/2010 18:34:38
The detector counts incoming microwave peaks. When it gets to 9,192,631,770 we say a second has elapsed. The frequency of the light is then 9,192,631,770 Hz by definition.


Actually, the detector does not count microwave peaks.

An oscillator produces microwave energy that causes the caesium to fluoresce. The detector measures the amount of fluorescence. Maximum fluorescence (photon emissions I suppose) is achieved when the microwave energy is tuned to 9.1xxx GHz. The clock makes very small adjustments to the oscillator to maintain maximum photon emission.

The clock is really comparing the natural oscillation of the caesium atom with the frequency of a microwave resonator and adjusting the resonator to match the frequency of the caesium. So, it's not really measuring a property of light at all. The caesium light output is simply used as a means to keep the resonator "in tune" with the caesium.
Title: Will a photon clock run at a different rate from an atomic clock under gravity?
Post by: graham.d on 22/05/2010 19:54:59
"The spatial change is radial length contraction, so when you hold your clocks flat you avoid this." - Farsight

If you note, Farsight, I positioned both clocks in a near zero gravitational field. The only difference is gravitational potential so I am not sure how the issue of orientation is relevant. I am not sure whether is is possible to determine whether there is length change or lightspeed lowering. A proper GR treatment would simply give the result that the spacetime interval was agreed by all observers. As I said previously, the remote measurement of time intervals for light travelling has to be thought out carefully. It is necessary to define the events of emission and detection with care.

You quote Baez who says "[...] a more modern interpretation is that the speed of light is constant in general relativity", but contradict this statement, so I don't follow your reasoning. You seem very definite but there seems varying opinions. If you read my lengthy description of a local and distant observer measuring the photon clock, I would be interested to know if you see anything wrong in the reasoning.

A light pulse going around lots of coils of fibre optic might be a practical photon clock too. With enough coils the time delay can be significant and measurable with low errors.
Title: Will a photon clock run at a different rate from an atomic clock under gravity?
Post by: amrit on 22/05/2010 22:36:46
RESUME

Relativistic effects of relative velocity of material change start above photon scale

Constancy of the light velocity in different inertial systems and areas of space with different gravity implies that relativistic effects of relative velocity of material change start on the scale above photon.

Special Theory of Relativity and General Theory of Relativity considers light moves through the space with constant velocity regardless upon the velocity of inertial system and strength of gravitation. This implies that at the scale of the photon and below at the scale of Planck relativistic effects of relative velocity of material change does not exist. Here is proposed that in all inertial systems velocity of a photon clock is unchanged. Change of gravity does not effect velocity of a photon clock as it effect velocity of an atom clock. Changes of atomic clocks are well known and calculated by Relativity in the Global Positioning System.

We have a “photon clock” made out of two mirrors A and B. Photon is moving from A to B, back to A and so on. One traveling of the photon between A and B is a “tick” of the clock. We take two photon clocks. One photon clock is on the surface of the earth, second is 4200 meters below at the bottom of the mine shaft. Velocity of light is invariant on gravity; both of clocks will “tick” with the same velocity.
We take two atomic clocks. One clock we put beside photon clock on the surface and second beside clock that is 4200 meters deep. According to the relativistic gravitational effect second atom clock will in 30 days “tick” faster as the atom clock on the surface for.

Photon clock will also have unchanged velocity in all different inertial systems, because velocity of light is equal in all different inertial systems. Similar experiment with 2 atom clocks an 2 photon clocks can be carried out by putting one atomic and one photon clock in the orbit station and one atom clock and one photon clock on the surface of the earth.

Special Relativity SR is postulated on constancy of light velocity. Equality of inertial and gravitational mass is connecting SR and General Relativity GR. Discussing on possibility that gravity infects velocity of light put under question relatedness of SR and GR. Gravitational red shift shows that gravity influences only frequency and not velocity of light. This means that “thought experiment” with photonic clock is correct: Velocity of photonic clock is invariant on gravity. In stronger gravity photon moving between mirror A and B change only frequency and not velocity. And this means that relativity gravitational effect of relative velocity of material change starts above photon scale.

Planck Clock
In this “Gedanken experiment” we use a photon clock that is made out of two mirrors that are on the distance of Planck. Photon moves from one to other mirror into a Planck time. This is so called “Planck clock”. Planck time is a fundament al unit to measure numerical order of material change i.e. motion in a timeless space. “Length dilatation” and “time dilatation” do not work on the Planck clock. Planck distance can not be smaller as it is. Planck distance is a fundamental constant invariant on the “length dilatation”. So by the “Planck clock” there is also no “time dilatation. Length dilatation and time dilatation exist above scale of the photon.

Invariance of light velocity excludes existence of relativistic effects of relative velocity of material change at the photon scale. Experiment with photon clocks and atomic clocks will give us more experimental data.
Title: Will a photon clock run at a different rate from an atomic clock under gravity?
Post by: Geezer on 23/05/2010 00:18:36
I'm probably missing something here, but velocity is a measurement of distance in time. Surely any measurement of velocity is therefore determined by the time local to that distance?

This does not appear to contradict the constant speed of light. In fact, if photons did not "observe" local time, would not the speed of light vary?
Title: Will a photon clock run at a different rate from an atomic clock under gravity?
Post by: amrit on 23/05/2010 07:05:38
photon moves in space only and not in time
c = Planck distance / Planck time
Planck time is a fundamental unit for measuring numerical order  of material change i.e. motion
Space-time is timeless, space-time is 4D.
see more in my article published in Physics Essays in last issue.
Title: Will a photon clock run at a different rate from an atomic clock under gravity?
Post by: Geezer on 23/05/2010 17:54:13
photon moves in space only and not in time

Relative to the photon I think that is true. Relative to us the photon does take time, therefore we observe velocity and we can confirm this experimentally in many different ways.
Title: Will a photon clock run at a different rate from an atomic clock under gravity?
Post by: amrit on 23/05/2010 21:53:25
By photon clock “time dilatation” is in contradiction with “length contraction”
   We have a photon clock in a fast airplane.  Mirrors are fixed in a way that photon moves along the direction of motion of the airplane. Distance between the mirrors is shortened by the length contraction. Because of the shorter distance between mirrors a path for the photon is shorter and so photon clock on the airplane “ticks” faster than photon clock on the surface of the earth. We know that atom clock in a fast airplane ticks slower than atom clock on the earth. Solution of this contradiction is in a preposition that photon clock in the airplane do not shorten. “Length contraction” is only a mathematical calculation that has no correspondence to the physical world. “Time dilatation” has also no correspondence in the physical world. What really happens by time dilatation is that velocity of material change velocity of clocks including slows down. Clocks run in space only and not in time
Title: Will a photon clock run at a different rate from an atomic clock under gravity?
Post by: amrit on 23/05/2010 21:55:51
photon moves in space only and not in time

Relative to the photon I think that is true. Relative to us the photon does take time, therefore we observe velocity and we can confirm this experimentally in many different ways.

No motion of photon is timeless, photon moves only in space. With clocks we measure numerical order of motion.
Title: Will a photon clock run at a different rate from an atomic clock under gravity?
Post by: amrit on 23/05/2010 22:07:04
Let’s take a photon that is moving on the distance d  between point A and point B of space. Distance  d is composed out of Planck distances dp1 + dp2 + dp3....dpn. The smallest distance a photon can pass on the way from A to B is dp . A unit of numerical order of photon motion from  dp1 to dp2  is the Planck time tp . The photon moves exclusively in space and not in time. The photon position dp1  is “before” position dp2  in a sense that the numerical order  n is “before” n+1 . Equivalently as the natural number 1 is “before” the natural number 2. Numerical order of material change is measured with the “ticking” of a clock where t0  represents the beginning of measurement, and  tn the end of measurement. Velocity v of a material change is derived from its numerical order:
v = d/tn. Frequency of material change is derived from its numerical order: Frequency = 1/tn
Title: Will a photon clock run at a different rate from an atomic clock under gravity?
Post by: graham.d on 23/05/2010 23:38:19
Amrit, this is just special relativity and there is no contradiction here. You are making a mistake in your calculations. You MUST measure time between events. The time of flight for the photon, from the perspective of the stationary observer, is not the same in each direction. If you imagine the photon is emitted (event 1) opposite to the motion direction from the front mirror, it reaches the back mirror more quickly because the back mirror is moving towards it. Work out the time to reach that mirror (event 2). After reflection it now has further to go to reach the front mirror which is receding from it. Work out the time it takes to get there (event 3). You will see that, from the perspective of the observer the time from event 1 to event 2 is shorter than event 2 to event 3. The total time is the sum of the two intervals. I am not going to do the maths here but it works out to be the 2 x the Lorentz contracted distance between the mirrors divided by the speed of light. This is exactly as SR predicts.

Nearly all apparant paradoxes in SR are because this sort of mistake is made, including the famous twin paradox.
Title: Will a photon clock run at a different rate from an atomic clock under gravity?
Post by: graham.d on 24/05/2010 09:02:55
Amrit, I thought I would show the maths anyway:

Let t1 be the first interval and t2 the second interval

c.t1 = d' - v.t1, c.t2 = d' + v.t2    where d' is the Lorentz Contracted mirror spacing

Hence t1 + t2 = d'/(c+v) + d'/(c-v)

              = (2d/c).[1/(1-v²/c²)]


Substituting the Lorentz contraction value for d'

d' = d.√(1-v²/c²)
where d is the proper mirror spacing gives

t1 + t2 = (2d/c).[1/√(1-v²/c²)]

Which shows that the stationary observer sees the photon clock running slow by the anount given by the cusomary time dilation equation.

Title: Will a photon clock run at a different rate from an atomic clock under gravity?
Post by: amrit on 24/05/2010 10:22:34
yes Graham, here is the point. What mathematic shows in not in accordance with experimental data. Light velocity is invariant on inertial system.
This simply means there is no length contraction in physical world.
Velocity of photon clock is invariant on inertial system as is case with C.

Title: Will a photon clock run at a different rate from an atomic clock under gravity?
Post by: Murchie85 on 24/05/2010 10:35:53
Isn't relativity and time dialation tested every single time we put a satellite into orbit and the fact the two actual clocks or time settings are adjusted to compensate for this?
Title: Will a photon clock run at a different rate from an atomic clock under gravity?
Post by: JP on 24/05/2010 10:39:18
Hi Amrit,

It sounds like what you're claiming is that special relativity as it's usually formulated has errors.  If that's the case, it's a new theory rather than mainstream physics.  Do you mind posting about it further in the New Theories section of the board?  

Thanks,

JP (moderator)
Title: Will a photon clock run at a different rate from an atomic clock under gravity?
Post by: amrit on 24/05/2010 11:04:08
Isn't relativity and time dialation tested every single time we put a satellite into orbit and the fact the two actual clocks or time settings are adjusted to compensate for this?

Yes different velocity of clocks is measured, not time. Space is timeless, space is not 3D + T, space is 4D, see my article on
http://scitation.aip.org/getabs/servlet/GetabsServlet?prog=normal&id=PHESEM000023000002000330000001&idtype=cvips&gifs=yes&ref=no
Title: Will a photon clock run at a different rate from an atomic clock under gravity?
Post by: amrit on 24/05/2010 11:06:40
Hi Amrit,

It sounds like what you're claiming is that special relativity as it's usually formulated has errors.  If that's the case, it's a new theory rather than mainstream physics.  Do you mind posting about it further in the New Theories section of the board?  

Thanks,

JP (moderator)

SR has no errors, X4 is not temporal, X4 is spatial too by the math formalism X4 = ict X4 is spatial distance ( d = v x t )where t means "tick" of clock in space that is timeless.
Title: Will a photon clock run at a different rate from an atomic clock under gravity?
Post by: graham.d on 24/05/2010 11:13:20
Amrit, the maths I used is based on Special Relativity and assumes lightspeed is invariant. You may argue your point regarding gravitational effects but Special Relativity is extremely well accepted and has yet to be contradicted by any experiment. In fact the concept of the photon clock is used in many undergraduate text books as a way of demonstrating how the invariance of lightspeed leads to the conclusion that there IS time dilation and Lorentz contraction.
Title: Will a photon clock run at a different rate from an atomic clock under gravity?
Post by: amrit on 24/05/2010 11:24:31
Graham,

“time dilatation” means only that the velocity of clock slows down.
Not as a result time running slower, because there is no time as a 4-th dimension of space.
Physical time is “ticking” if clock in space that itself is timeless.
Velocity of photon clock is equal in all inertial systems because light has same speed in all inertial systems.
More than that:
By photon clock “time dilatation” is in contradiction with “length contraction”
   We have a photon clock in a fast airplane.  Mirrors are fixed in a way that photon moves along the direction of motion of the airplane. Distance between the mirrors is shortened by the length contraction. Because of the shorter distance between mirrors a path for the photon is shorter and so photon clock on the airplane “ticks” faster than same construction photon clock on the surface of the earth.
Experimental data show that atom clock in a fast airplane ticks slower than atom clock on the earth. Solution of this contradiction is in a preposition that photon clock in the airplane do not shorten. “Length contraction” is only a mathematical calculation that has no correspondence to the physical world. “Time dilatation” has also no correspondence in the physical world. What really happens by “time dilatation” is that velocity of material change velocity of clocks including slows down. Material change clocks run included run in space only and not in time.

Experimental data shows that velocity of atom clocks change. So relativity of velocity of material change starts above photon scale.

See my last article:
http://www.fqxi.org/data/forum-attachments/Relativistic_effects_of_felative__velocity__vixra.pdf
Title: Will a photon clock run at a different rate from an atomic clock under gravity?
Post by: graham.d on 24/05/2010 11:30:12
By photon clock “time dilatation” is in contradiction with “length contraction”
   We have a photon clock in a fast airplane.  Mirrors are fixed in a way that photon moves along the direction of motion of the airplane. Distance between the mirrors is shortened by the length contraction.
OK

Quote
Because of the shorter distance between mirrors a path for the photon is shorter and so photon clock on the airplane “ticks” faster than photon clock on the surface of the earth.


No. Firstly the photon has a short path because the mirror is approaching it but then it has a long path because the front mirror is receding from it. As the plane approaches lightspeed this long path dominates and the time to get there will tend to infinity. The length contracts, but not so much that the time delay for the photon to travel in both directions gets shorter. See the maths I took the time to work out for you. The time for the photon travel is not the same in each direction.
Quote
We know that atom clock in a fast airplane ticks slower than atom clock on the earth.
Yes, and there is no contradiction as I have shown.
Title: Will a photon clock run at a different rate from an atomic clock under gravity?
Post by: amrit on 24/05/2010 11:38:30
By photon clock “time dilatation” is in contradiction with “length contraction”
   We have a photon clock in a fast airplane.  Mirrors are fixed in a way that photon moves along the direction of motion of the airplane. Distance between the mirrors is shortened by the length contraction.
OK

Quote
Because of the shorter distance between mirrors a path for the photon is shorter and so photon clock on the airplane “ticks” faster than photon clock on the surface of the earth.


No. Firstly the photon has a short path because the mirror is approaching it but then it has a long path because the front mirror is receding from it. As the plane approaches lightspeed this long path dominates and the time to get there will tend to infinity. The length contracts, but not so much that the time delay for the photon to travel in both directions gets shorter. See the maths I took the time to work out for you. The time for the photon travel is not the same in each direction.
Quote
We know that atom clock in a fast airplane ticks slower than atom clock on the earth.


Yes, and there is no contradiction as I have shown.



Graham, in SR length is getting shorter into direction of motion. Suppose our photon clock is 5 cm long. Because of the “length contraction” our clock will shrink a bit.
So is will “tick” faster.
But we know atom clocks run slower.
So there is a mistake here. Solution is: C is constant and there is no length contraction in material universe
Title: Will a photon clock run at a different rate from an atomic clock under gravity?
Post by: graham.d on 24/05/2010 11:54:07
Amrit you are missing the point. You have to follow the events very carefully. The light gets from the front to the back mirror quickly because the mirror is coming towards it (time t1 in the maths). But it then takes a long time to get to the front mirror which is receding from it (time t2). The sum of these two times (t1+t2) is longer, as seen from the stationary observer, than the proper time (observer on the plane). Even though the plane is Lorentz contracted the net result is still that the time is longer. You have to follow the maths carefully. If you can find an error then please point it out.
Title: Will a photon clock run at a different rate from an atomic clock under gravity?
Post by: Farsight on 24/05/2010 12:24:25
If you note, Farsight, I positioned both clocks in a near zero gravitational field. The only difference is gravitational potential so I am not sure how the issue of orientation is relevant.
In SR the length contraction is in the direction of motion, and in GR via the principle of equivalence it's radial. The local strength of the gravitational field indicates the rate of change of gravitational potential at this location.

I am not sure whether is is possible to determine whether there is length change or lightspeed lowering. A proper GR treatment would simply give the result that the spacetime interval was agreed by all observers. As I said previously, the remote measurement of time intervals for light travelling has to be thought out carefully. It is necessary to define the events of emission and detection with care.
It's tricky to say what the proper GR treatment is. When you read the original it is different to what's in modern textbooks, and that brings us back to the conflict between interpretations again.  

You quote Baez who says "[...] a more modern interpretation is that the speed of light is constant in general relativity", but contradict this statement, so I don't follow your reasoning.
Einstein's interpretation was that the speed of light varies, but the modern interpretation is that it's constant. I side firmly with Einstein because IMHO this is backed up by evidence like the Shapiro delay.

You seem very definite but there seems varying opinions.
I am. Einstein started with the constant speed of light as a postulate in 1905, but in 1911 he wrote On the Influence of Gravitation on the Propagation of Light (http://www.relativitybook.com/resources/Einstein_gravity.html), where he gives the expression c = c0(1 + Φ/c²). Here c is varying with gravitational potential. Then in 1912 he said "On the other hand I am of the view that the principle of the constancy of the velocity of light can be maintained only insofar as one restricts oneself to spatio-temporal regions of constant gravitational potential". He repeated this in 1913 when he said this: "I arrived at the result that the velocity of light is not to be regarded as independent of the gravitational potential. Thus the principle of the constancy of the velocity of light is incompatible with the equivalence hypothesis". There it is again in 1915 when he says " the writer of these lines is of the opinion that the theory of relativity is still in need of generalization, in the sense that the principle of the constancy of the velocity of light is to be abandoned."  That’s on page 259 of Doc 21, sorry, I’m not sure what the original paper is called. He says it again in late 1915, on page 150 of Doc 30, within The Foundation of the General Theory of Relativity (http://www.alberteinstein.info/gallery/pdf/CP6Doc30_English_pp146-200.pdf). Einstein says "the principle of the constancy of the velocity of light in vacuo must be modified.". He spells it out in section 22 of the 1916 book Relativity: The Special and General Theory (http://www.gutenberg.org/etext/5001) where he says this:

"In the second place our result shows that, according to the general theory of relativity, the law of the constancy of the velocity of light in vacuo, which constitutes one of the two fundamental assumptions in the special theory of relativity and to which we have already frequently referred, cannot claim any unlimited validity. A curvature of rays of light can only take place when the velocity of propagation of light varies with position. Now we might think that as a consequence of this, the special theory of relativity and with it the whole theory of relativity would be laid in the dust..."

People see the word velocity in the translations without seeing the context and without noticing that he’s repeatedly referring to “the principle” and "one of the two fundamental assumptions". That's the SR postulate, which is the constant speed of light. And it's important to note that what he actually said was in German. it was die Ausbreitungsgeschwindigkeit des Lichtes mit dem Orte variiert. It translates into the speed of light varies with the locality. The word “velocity” in the 1920 Methuen translation was the common usage, as in “high velocity bullet”, not the vector quantity that combines speed and direction. He was saying the speed varies with position, and it causes the curvilinear motion. It causes the light to follow a curved path, like a car veers when the near-side wheels encounter mud at the side of the road.

If you read my lengthy description of a local and distant observer measuring the photon clock, I would be interested to know if you see anything wrong in the reasoning.
I read your post (http://www.thenakedscientists.com/forum/index.php?topic=31723.msg310370#msg310370). What's wrong with it is that the Lorentz contraction in SR corresponds to the radial length contraction in GR. If you say your distant observer up in space isn't moving with respect to your observer at the centre of the earth, and if both observers hold their clocks flat, there is no length change. You can then remove redshift with a single observer who leaves a light-clock in space and takes another synchronised light-clock down to the centre of the earth for a while. When he goes back up into space the clocks are no longer synchronised. So the speed must be different.    

A light pulse going around lots of coils of fibre optic might be a practical photon clock too. With enough coils the time delay can be significant and measurable with low errors.
Sounds good to me graham.
Title: Will a photon clock run at a different rate from an atomic clock under gravity?
Post by: amrit on 24/05/2010 12:28:52
Amrit you are missing the point. You have to follow the events very carefully. The light gets from the front to the back mirror quickly because the mirror is coming towards it (time t1 in the maths). But it then takes a long time to get to the front mirror which is receding from it (time t2). The sum of these two times (t1+t2) is longer, as seen from the stationary observer, than the proper time (observer on the plane). Even though the plane is Lorentz contracted the net result is still that the time is longer. You have to follow the maths carefully. If you can find an error then please point it out.

Graham

GPR corrections of velocity are equal for all observers.
There is no such a thing as “proper time”.
There is only a change of velocity of clock on the orbit that is valid for the observer on the orbit and for the observer on the surface of the earth.

In classical example of SR clock run slower on the fast train for the observer on the station and for the observer in the train. This is what shows experimental data.

In Relativity is relative velocity of material change and velocity of clocks and is valid for all observers.  Observer is not even necessary. Clock runs slower on the orbit without watching it.

Relativity is a very nature of the universe. Just it starts above photon size. Constancy of c point that out clearly.

Yours amrit
PS read my article on
http://www.vetrnica.net/index.php?option=com_content&view=article&id=21&Itemid=23
Title: Will a photon clock run at a different rate from an atomic clock under gravity?
Post by: amrit on 24/05/2010 12:34:32
If you note, Farsight, I positioned both clocks in a near zero gravitational field. The only difference is gravitational potential so I am not sure how the issue of orientation is relevant.
In SR the length contraction is in the direction of motion, and in GR via the principle of equivalence it's radial. The local strength of the gravitational field indicates the rate of change of gravitational potential at this location.

I am not sure whether is is possible to determine whether there is length change or lightspeed lowering. A proper GR treatment would simply give the result that the spacetime interval was agreed by all observers. As I said previously, the remote measurement of time intervals for light travelling has to be thought out carefully. It is necessary to define the events of emission and detection with care.
It's tricky to say what the proper GR treatment is. When you read the original it is different to what's in modern textbooks, and that brings us back to the conflict between interpretations again.  

You quote Baez who says "[...] a more modern interpretation is that the speed of light is constant in general relativity", but contradict this statement, so I don't follow your reasoning.
Einstein's interpretation was that the speed of light varies, but the modern interpretation is that it's constant. I side firmly with Einstein because IMHO this is backed up by evidence like the Shapiro delay.

You seem very definite but there seems varying opinions.
I am. Einstein started with the constant speed of light as a postulate in 1905, but in 1911 he wrote On the Influence of Gravitation on the Propagation of Light (http://www.relativitybook.com/resources/Einstein_gravity.html), where he gives the expression c = c0(1 + Φ/c²). Here c is varying with gravitational potential. Then in 1912 he said "On the other hand I am of the view that the principle of the constancy of the velocity of light can be maintained only insofar as one restricts oneself to spatio-temporal regions of constant gravitational potential". He repeated this in 1913 when he said this: "I arrived at the result that the velocity of light is not to be regarded as independent of the gravitational potential. Thus the principle of the constancy of the velocity of light is incompatible with the equivalence hypothesis". There it is again in 1915 when he says " the writer of these lines is of the opinion that the theory of relativity is still in need of generalization, in the sense that the principle of the constancy of the velocity of light is to be abandoned."  That’s on page 259 of Doc 21, sorry, I’m not sure what the original paper is called. He says it again in late 1915, on page 150 of Doc 30, within The Foundation of the General Theory of Relativity (http://www.alberteinstein.info/gallery/pdf/CP6Doc30_English_pp146-200.pdf). Einstein says "the principle of the constancy of the velocity of light in vacuo must be modified.". He spells it out in section 22 of the 1916 book Relativity: The Special and General Theory (http://www.gutenberg.org/etext/5001) where he says this:

"In the second place our result shows that, according to the general theory of relativity, the law of the constancy of the velocity of light in vacuo, which constitutes one of the two fundamental assumptions in the special theory of relativity and to which we have already frequently referred, cannot claim any unlimited validity. A curvature of rays of light can only take place when the velocity of propagation of light varies with position. Now we might think that as a consequence of this, the special theory of relativity and with it the whole theory of relativity would be laid in the dust..."

People see the word velocity in the translations without seeing the context and without noticing that he’s repeatedly referring to “the principle” and "one of the two fundamental assumptions". That's the SR postulate, which is the constant speed of light. And it's important to note that what he actually said was in German. it was die Ausbreitungsgeschwindigkeit des Lichtes mit dem Orte variiert. It translates into the speed of light varies with the locality. The word “velocity” in the 1920 Methuen translation was the common usage, as in “high velocity bullet”, not the vector quantity that combines speed and direction. He was saying the speed varies with position, and it causes the curvilinear motion. It causes the light to follow a curved path, like a car veers when the near-side wheels encounter mud at the side of the road.

If you read my lengthy description of a local and distant observer measuring the photon clock, I would be interested to know if you see anything wrong in the reasoning.
I read your post (http://www.thenakedscientists.com/forum/index.php?topic=31723.msg310370#msg310370). What's wrong with it is that the Lorentz contraction in SR corresponds to the radial length contraction in GR. If you say your distant observer up in space isn't moving with respect to your observer at the centre of the earth, and if both observers hold their clocks flat, there is no length change. You can then remove redshift with a single observer who leaves a light-clock in space and takes another synchronised light-clock down to the centre of the earth for a while. When he goes back up into space the clocks are no longer synchronised. So the speed must be different.    

A light pulse going around lots of coils of fibre optic might be a practical photon clock too. With enough coils the time delay can be significant and measurable with low errors.
Sounds good to me graham.

Farsight gravitational red shift shows light velocity c is constant. Only frequency changes. If c is not constant connection betwen SR and GR is broken. This would be a bit to much....I do not think in Relativity there is a mistake....Relativity is perfect. The case is that there is no time there. With clocks we measure numerical order only...
http://www.vetrnica.net/index.php?option=com_content&view=article&id=21&Itemid=23
yours amrit
Title: Will a photon clock run at a different rate from an atomic clock under gravity?
Post by: Farsight on 24/05/2010 12:39:35
An oscillator produces microwave energy that causes the caesium to fluoresce. The detector measures the amount of fluorescence. Maximum fluorescence (photon emissions I suppose) is achieved when the microwave energy is tuned to 9.1xxx GHz. The clock makes very small adjustments to the oscillator to maintain maximum photon emission.
Noted Geezer, but remember that this is defining the second. Hertz is cycles per second, so we're finding the maximum then defining the frequency to be 9.192631770 GHz.

The clock is really comparing the natural oscillation of the caesium atom with the frequency of a microwave resonator and adjusting the resonator to match the frequency of the caesium. So, it's not really measuring a property of light at all.
The oscillation is a hyperfine transition, and electron spin-flip. The event is electromagnetic, as is the emitted light. If the gravitational potential is lower, all electromagnetic phenomena occur at a slower rate. We call it time dilation, but that rather misses the obvious fact that electromagnetic spin flips are happening slower and the emitted light is moving slower too.
Title: Will a photon clock run at a different rate from an atomic clock under gravity?
Post by: Farsight on 24/05/2010 12:58:17
Farsight gravitational red shift shows light velocity c is constant. Only frequency changes. If c is not constant connection betwen SR and GR is broken.
This isn't true I'm afraid, amrit. The frequency of the light isn't relevant. Start with two synchronised light clocks, leave one up in space, take the other down to the planet for a while taking care to avoid radial length contraction, then take it back up to space. The two clocks are no longer synchronised. Yes, the local speed of light is always measured to be 299,792,458 metres per second, but those two different clock readings tell you that 299,792,458 metres per second up in space is not the same as 299,792,458 metres per second down on the planet. People say the difference is because of "time dilation", but you and I know that clocks clock up motion rather than "the flow of time". Hence we know that a light clock clocks up the motion of light. Those two different readings are crystal-clear evidence that down on the planet, the light goes slower.   
Title: Will a photon clock run at a different rate from an atomic clock under gravity?
Post by: amrit on 24/05/2010 13:01:41
An oscillator produces microwave energy that causes the caesium to fluoresce. The detector measures the amount of fluorescence. Maximum fluorescence (photon emissions I suppose) is achieved when the microwave energy is tuned to 9.1xxx GHz. The clock makes very small adjustments to the oscillator to maintain maximum photon emission.
Noted Geezer, but remember that this is defining the second. Hertz is cycles per second, so we're finding the maximum then defining the frequency to be 9.192631770 GHz.

The clock is really comparing the natural oscillation of the caesium atom with the frequency of a microwave resonator and adjusting the resonator to match the frequency of the caesium. So, it's not really measuring a property of light at all.
The oscillation is a hyperfine transition, and electron spin-flip. The event is electromagnetic, as is the emitted light. If the gravitational potential is lower, all electromagnetic phenomena occur at a slower rate. We call it time dilation, but that rather misses the obvious fact that electromagnetic spin flips are happening slower and the emitted light is moving slower too.

Farsight,

I’m not expert for clocks. As far as I know atomic clock works on atom frequency. Photon clock works on photon motion in space. Difference is quite clear. I’m emotionally tuned with c as a constant. It has its own elegance this idea. As far no experiment will prove opposite I stay with it.

Title: Will a photon clock run at a different rate from an atomic clock under gravity?
Post by: amrit on 24/05/2010 13:03:27
Farsight gravitational red shift shows light velocity c is constant. Only frequency changes. If c is not constant connection betwen SR and GR is broken.
This isn't true I'm afraid, amrit. The frequency of the light isn't relevant. Start with two synchronised light clocks, leave one up in space, take the other down to the planet for a while taking care to avoid radial length contraction, then take it back up to space. The two clocks are no longer synchronised. Yes, the local speed of light is always measured to be 299,792,458 metres per second, but those two different clock readings tell you that 299,792,458 metres per second up in space is not the same as 299,792,458 metres per second down on the planet. People say the difference is because of "time dilation", but you and I know that clocks clock up motion rather than "the flow of time". Hence we know that a light clock clocks up the motion of light. Those two different readings are crystal-clear evidence that down on the planet, the light goes slower.  

Farsight I do not buy that.
yours amrit
Title: Will a photon clock run at a different rate from an atomic clock under gravity?
Post by: graham.d on 24/05/2010 13:06:23
Amrit, "proper time" has a specific definition to be just that of an observer comoving with the moving frame. Your link does not work properly by the way.

I appreciate English is not your first language, but however hard I try, I cannot understand the sense of what you are saying. Perhaps you could show me where my maths is at fault?
Title: Will a photon clock run at a different rate from an atomic clock under gravity?
Post by: Farsight on 24/05/2010 13:49:57
We'll have to agree to differ, amrit. Sorry I couldn't help.
Title: Will a photon clock run at a different rate from an atomic clock under gravity?
Post by: graham.d on 24/05/2010 15:36:21
Farsight, I understand Einstein's view on this but note he was uncomfortable with this view. And I take your point about the clocks being "flat" which I take to mean parallel to a circumference as opposed to in-line and parallel to a radius. I don't think you can conclude that this removes the difference that may occur between remote length measurements although I admit that the maths is sufficiently hard that I could not show it would produce the same quantitative result. For example you could imagine a distant observer receiving signals from the mirrors at each end of the apparatus which were set up to send beams outward exactly parallel, but because they are in a divergent field they will follow geodesics that would have them diverge such that when they reached the distant observer they would show a bigger gap between the mirrors. Now it is very hard to say that this would yield the same value as the simpler calculation for the mirrors in line, but is, nonetheless, possible. The maths to calculate this is a bit hard.

I will give this a little more thought. A tenet of GR is that the all observers should agree on the spacetime interval. I am not sure that it is necessarily possible to resolve the meaning in term of just space and just time in this case. As the modern view is, as you say, that lightspeed is constant, how would these physicist view this scenario?
Title: Will a photon clock run at a different rate from an atomic clock under gravity?
Post by: imatfaal on 24/05/2010 16:20:14

A light pulse going around lots of coils of fibre optic might be a practical photon clock too. With enough coils the time delay can be significant and measurable with low errors.

3 coils at 90 degrees to each other would nicely (after serious number crunching) allow dilation from varying gravitational potential and dilation from relative velocities to be separated - I think...
Title: Will a photon clock run at a different rate from an atomic clock under gravity?
Post by: JP on 24/05/2010 16:24:01
Actually, I'm not sure that there's any contradiction between the idea that the speed of light might not seem constant over large distances and the speed of light being constant locally in a vacuum.  From what I understand of it, you can think of it like traveling on the earth.  Say it takes me 4 hours to travel from Paris to Los Angeles.  I could calculate my average speed by taking the distance between them in 3D space (which is the length straight line through the globe divided by 4 hours) or I could measure my speed at every instant as I fly around the globe and average that.  The path through the globe will be far shorter than the actual speed I was traveling.  I think it's a similar case with GR and the speed of light.  If you look at lengthy paths and just try to calculate the speed of light based on what you'd expect if space-time were flat and the light moves at c, you find that it takes longer.  However, if you look at the light at every instant, it's moving at the speed c--it just takes a curved path, so the total time is longer.

Or put another way, if you look locally enough so that you're calculating instantaneous speed, space-time is flat and SR holds.  It's when you look over large enough regions of curved space-times that you get these deviations.  (I guess even if you're comparing two flat regions--which is what this discussion is about--measurements can differ because space-time can be stretched, but not curved.  Using the rulers and clocks of one region to measure the other will also cause the speed of light to appear to be slowed.)  In fact, I think you can go further and say that if the speed of light wasn't constant locally, the geometrical interpretation of general relativity would fail.  

Is there an equally valid model that allows the speed of light to vary locally and produces identical results (at least within the regimes we've tested)?  I don't know.  
Title: Will a photon clock run at a different rate from an atomic clock under gravity?
Post by: graham.d on 24/05/2010 17:08:14
Paris to LA in 4 hours - I didn't think the Ramjet was flying yet!

It is interesting to think of two regions that are both essentially flat regions of Minkowski space where they have different degrees of Stretched Space-Time. There would seem to be no way of determining the degree of "Stretchedness" from inside such a region because all the physical laws would work fine in both regions. So if one region is open to investigation from the other (as in the case postulated previously) it seems odd that the observed behaviour would depend on the spacetime between the two regions. I think this is what you are saying JP. I could easily imagine that you could measure the temporal and spatial difference but it is hard to believe that these would be different depending on the route your information traveled to get to you.

I think that Farsight was alluding to the idea that the velocity of light being a tensor field (rather than a scalar field) in having different radial and tangential values in a schwartzchild metric. It still seems odd that the intervening space would affect what should be scalar measurements in flat space though. Any ideas? I may be missing some key points here.
Title: Will a photon clock run at a different rate from an atomic clock under gravity?
Post by: Farsight on 24/05/2010 18:45:00
Farsight, I understand Einstein's view on this but note he was uncomfortable with this view.
It's an interesting one Graham. One day when the dust has settled somebody will write an authorative narrative on the long and tortuous trail, and it will be fascinating reading.

And I take your point about the clocks being "flat" which I take to mean parallel to a circumference as opposed to in-line and parallel to a radius.
Thanks, sorry if I didn't make it clear.   

I don't think you can conclude that this removes the difference that may occur between remote length measurements although I admit that the maths is sufficiently hard that I could not show it would produce the same quantitative result.
I think you can actually, very simply. If you go along with GR radial length contraction, and Einstein, and the Occam's razor direct evidence that says the light is moving slower rather than "time runs slower", you can see it. The metre is defined as "the distance travelled by light in a complete vacuum in 1⁄299,792,458th of a second". Hence the bigger second and the slower light cancel each other out. The metre is unchanged.   

For example you could imagine a distant observer receiving signals from the mirrors at each end of the apparatus which were set up to send beams outward exactly parallel, but because they are in a divergent field they will follow geodesics that would have them diverge such that when they reached the distant observer they would show a bigger gap between the mirrors. Now it is very hard to say that this would yield the same value as the simpler calculation for the mirrors in line, but is, nonetheless, possible. The maths to calculate this is a bit hard.
Sorry, I'm not keen on this because it seems to go against the grain of general relativity regardless of which interpretation one prefers. 

I will give this a little more thought. A tenet of GR is that the all observers should agree on the spacetime interval. I am not sure that it is necessarily possible to resolve the meaning in term of just space and just time in this case. As the modern view is, as you say, that lightspeed is constant, how would these physicist view this scenario?
I'm not sure graham. But I'd say that to fix the speed of light as an absolute constant, one has to elevate time to something above and beyond the experimental and observational evidence, and say "time runs slower". The trouble is, when you look at a clock, you don't see time running at all. All you see is cogs moving, or a crystal oscillating, or a caesium atom oscillating, or light moving. It always comes down to motion one way or another. Clocks clock up motion, that's all they ever do, that's why in The Foundation of the General Theory of relativity Einstein gave us the equations of motion. And when the clock runs slower, it isn't really time running slower, it's the motion going slower. Have a google on "Farsight" and "Time Explained" for my take on this.
Title: Will a photon clock run at a different rate from an atomic clock under gravity?
Post by: Farsight on 24/05/2010 19:03:33
..I think that Farsight was alluding to the idea that the velocity of light being a tensor field (rather than a scalar field) in having different radial and tangential values in a schwartzchild metric.
I wasn't, I was just trying to keep things simple and focus on what we actually observe, which is two different readings from two identical parallel-mirror light-clocks at different locations where the gμv gravitational potential is different. 

It still seems odd that the intervening space would affect what should be scalar measurements in flat space though. Any ideas? I may be missing some key points here.
The intervening space isn't relevant. You could run the clocks for years and see the discrepancy grow ever larger. Redshift and signal time becomes increasingly inconsequential. Like you say, the spacetime is flat at both locations. There is no discernible curvature out in space or at the centre of the earth, because there is no detectable gμv gradient. The different measurements are because the space itself is different at those two locations.
Title: Will a photon clock run at a different rate from an atomic clock under gravity?
Post by: Geezer on 24/05/2010 19:37:41
An oscillator produces microwave energy that causes the caesium to fluoresce. The detector measures the amount of fluorescence. Maximum fluorescence (photon emissions I suppose) is achieved when the microwave energy is tuned to 9.1xxx GHz. The clock makes very small adjustments to the oscillator to maintain maximum photon emission.
Noted Geezer, but remember that this is defining the second. Hertz is cycles per second, so we're finding the maximum then defining the frequency to be 9.192631770 GHz.

The clock is really comparing the natural oscillation of the caesium atom with the frequency of a microwave resonator and adjusting the resonator to match the frequency of the caesium. So, it's not really measuring a property of light at all.
The oscillation is a hyperfine transition, and electron spin-flip. The event is electromagnetic, as is the emitted light. If the gravitational potential is lower, all electromagnetic phenomena occur at a slower rate. We call it time dilation, but that rather misses the obvious fact that electromagnetic spin flips are happening slower and the emitted light is moving slower too.

Er, well the second is defined as the interval for a certain number of the atomic events. The number was chosen to line up with more traditional definitions for the second. As you point out, the fundamental "tick" is an electromagnetic event with the atom.

The light output could be delayed in a very variable fashion, but it would have no effect on the frequency of the microwave resonator. The light output is only sampled periodically to make very infrequent and very minute adjustments to the frequency of the microwave resonator. I believe the detector only responds to the number of photons that reach it. I'm not sure how they know to adjust up or down because I would assume that the light output will diminish if the resonator is slow or fast relative to the caesium events.

Being pedantic, you can't really say that the events are happening at a "slower rate", because that seems to suggest that time itself is invariant.
Title: Will a photon clock run at a different rate from an atomic clock under gravity?
Post by: graham.d on 24/05/2010 22:58:21
Both spaces are flat Minkowski space and I think we are assuming that the physical laws are the same, as measured locally, at each location and that it would be expected that the gravitational potential is purely relative and has no meaning in absolute terms. Are these correct assumptions to work with? If, then, the only difference is that space is "stretched" more in one region than the other then this "scaling" must apply to (x,y,x,-ct) equally mustn't it? If not what coordinate transformation would apply? In SR the basis for the Lorentz transformation is the constancy of c to all observers. Is the only effect of gravitational potential (Φ) that c is a function of Φ and that this would be the basis of any coordinate transformation?

This seems reasonable but is it all consistent with observation or gedanken experiments?
Title: Will a photon clock run at a different rate from an atomic clock under gravity?
Post by: JP on 24/05/2010 23:54:45
Paris to LA in 4 hours - I didn't think the Ramjet was flying yet!

Heh.  I'm riding in my photon jet.  It would be faster, but it slowed down a bit because it's so close to the earth.
 [;)]
Title: Will a photon clock run at a different rate from an atomic clock under gravity?
Post by: Geezer on 25/05/2010 03:49:03


Heh.  I'm riding in my photon jet.  It would be faster, but it slowed down a bit because it's so close to the earth.
 [;)]


Yes, but I'm sure you'll arrive in no time at all.
Title: Will a photon clock run at a different rate from an atomic clock under gravity?
Post by: amrit on 25/05/2010 05:17:00
Amrit, "proper time" has a specific definition to be just that of an observer comoving with the moving frame. Your link does not work properly by the way.

I appreciate English is not your first language, but however hard I try, I cannot understand the sense of what you are saying. Perhaps you could show me where my maths is at fault?

Graham with your math all is fine. Just be aware clocks tick in space only and not in time. Time dilatation means that clocks run slower in a timeless space. You think in math terms, think in physical terms and you will discover universe is timeless. In the universe there is only motion, time belongs to the mind. Time is a mind frame through which we experience motion, read ma article below:
http://www.vetrnica.net/index.php?option=com_content&view=article&id=21&Itemid=23
yours amrit
Title: Will a photon clock run at a different rate from an atomic clock under gravity?
Post by: Geezer on 25/05/2010 06:23:37
In the universe there is only motion, time belongs to the mind. Time is a mind frame through which we experience motion, read ma article below:

Amrit,

I have tried to persuade the molecules that make up my body that time is only a state of mind, but they won't listen.

Isn't it high time we move this topic to New Theories?

Title: Will a photon clock run at a different rate from an atomic clock under gravity?
Post by: amrit on 25/05/2010 06:30:57
NO. This is not new theory. This is about right understanding of SR. We live in space only. X4 is not time, X4 is spatial too.

PS
missing understanding is that in SR velocity of clocks in all inertial systems is invariant of the observers. Observer has no influence on velocity of clocks. Clocks run with the same velocity for all observers.
Title: Will a photon clock run at a different rate from an atomic clock under gravity?
Post by: Geezer on 25/05/2010 06:51:09
Clocks run with the same velocity for all observers.

Yes. I completely agree with you there. As I see it, all things are affected by their local time. That means everything from subatomic events to fancy atomic clocks have to obey local time. Consequently, any observation of velocity (which can only be expressed in terms of time) will be the same for all observers.
Title: Will a photon clock run at a different rate from an atomic clock under gravity?
Post by: JP on 25/05/2010 07:20:36
Clocks run with the same velocity for all observers.

Yes. I completely agree with you there. As I see it, all things are affected by their local time. That means everything from subatomic events to fancy atomic clocks have to obey local time. Consequently, any observation of velocity (which can only be expressed in terms of time) will be the same for all observers.

I'm going to disagree just to be difficult.  But seriously, clocks don't measure velocity.  The "velocity of a clock" is how fast it goes when you throw it.  Clocks measure time.  If clocks only measured velocity, they couldn't count time, since velocity only tells you the ratio of distance to time.  You need a value for distance and for velocity to get time.  That's how light clocks work: by knowing the distance between the mirrors and using the known value of the speed of light. 

That's also why this theory doesn't agree with special relativity.  The velocity of light might be constant for all clocks, but their distance measurements don't agree, so they can't possibly tick at the same speed.
Title: Will a photon clock run at a different rate from an atomic clock under gravity?
Post by: Geezer on 25/05/2010 08:32:39
I'm going to disagree just to be difficult.  But seriously, clocks don't measure velocity.  The "velocity of a clock" is how fast it goes when you throw it.  Clocks measure time.

Far be it from me to disagree with someone who obviously knows what they are talking about, but clocks don't measure time. The only thing that clocks do is count events. The events may be determined by time, but I'm not sure that's quite the same thing as saying that clocks measure time.

Time measures clocks might be a better way to look at it.
Title: Will a photon clock run at a different rate from an atomic clock under gravity?
Post by: JP on 25/05/2010 09:05:27
I'm going to disagree just to be difficult.  But seriously, clocks don't measure velocity.  The "velocity of a clock" is how fast it goes when you throw it.  Clocks measure time.

Far be it from me to disagree with someone who obviously knows what they are talking about, but clocks don't measure time. The only thing that clocks do is count events. The events may be determined by time, but I'm not sure that's quite the same thing as saying that clocks measure time.

Time measures clocks might be a better way to look at it.

This is somewhat speculative, since I'm not a relativity expert:

I guess maybe since we're stuck using the language of special relativity here, maybe the best any device can do is to measure the interval between two events?  (The interval being Δx2-Δt2, where Δx is the spatial separation of the events and Δt is their temporal separation [with c=1]).  You can then get time by relating this interval to physical processes.  If your clock is using a cesium atom at rest, Δx=0, so your measurement of a "tick" is directly getting time out.  If your clock is a light clock, then you're measuring an interval which includes both a Δx and a Δt (I suspect you need to calculate two intervals: one in each direction, as graham.d did).  That's why you need to also use the fact that light speed is constant--which tells you that the interval between the events is zero, which in turn lets you calculate the time of a second from the distance between mirrors. 

---------------------------------
Now that I'm done confusing myself, this part is my real point:

The point I was trying to get across is that clocks spit out "time" as an answer, no matter what they're using as input.  Velocity isn't enough information on its own to give you time as an answer--you need to know a distance measurement as well, which is why you will still get time dilation in a photon clock even with a constant speed of light.

I guess a good question is whether or not a "good" clock (meaning one that measures local time accurately) is in practice any different than measuring time itself.  I don't know the answer, but I've been assuming that it isn't.
Title: Will a photon clock run at a different rate from an atomic clock under gravity?
Post by: Farsight on 25/05/2010 11:06:43
Er, well the second is defined as the interval for a certain number of the atomic events. The number was chosen to line up with more traditional definitions for the second. As you point out, the fundamental "tick" is an electromagnetic event with the atom.
And if the space is different, the underlying motion associated with this event is slower. It's a spin-flip, the electron is turning over. You can get a feel for this by stretching an eleastic band over your finger and thumb. Stick a pencil in, and turn the pencil over 180 degrees tensioning the elastic, then let go. It vibrates back and forth.   

The light output could be delayed in a very variable fashion, but it would have no effect on the frequency of the microwave resonator. The light output is only sampled periodically to make very infrequent and very minute adjustments to the frequency of the microwave resonator. I believe the detector only responds to the number of photons that reach it. I'm not sure how they know to adjust up or down because I would assume that the light output will diminish if the resonator is slow or fast relative to the caesium events.
It makes no odds, electromagnetic phenomena propagate at a slower rate in a region of lower gravitational potential.

Being pedantic, you can't really say that the events are happening at a "slower rate", because that seems to suggest that time itself is invariant.
Things move slower, that's all. When the motion is cyclic like the spin-flip, we call the result an event.
Title: Will a photon clock run at a different rate from an atomic clock under gravity?
Post by: Farsight on 25/05/2010 11:26:06
Both spaces are flat Minkowski space and I think we are assuming that the physical laws are the same, as measured locally, at each location and that it would be expected that the gravitational potential is purely relative and has no meaning in absolute terms. Are these correct assumptions to work with?
Yes, both spaces are flat and local physical laws are the same. But whilst that  gravitational potential is relative, it does have an absolute meaning. It's saying something pretty fundamental about the nature of space at that location, and relative measurements tell you that the difference between locations is very real.   

If, then, the only difference is that space is "stretched" more in one region than the other then this "scaling" must apply to (x,y,x,-ct) equally mustn't it? If not what coordinate transformation would apply?
I wouldn't say space is "stretched". Instead it's "squashed" in the radial direction, and we're trying to set that aside to make it clear that the -ct term has changed. The best colloquial term I can find is that space is "stronger" in a region of lower gravitational potential.

In SR the basis for the Lorentz transformation is the constancy of c to all observers. Is the only effect of gravitational potential (Φ) that c is a function of Φ and that this would be the basis of any coordinate transformation?
Not quite, because of the radial length contraction that equates to the SR length contraction. The effect of gravitational potential on c is the one to home in on. 

This seems reasonable but is it all consistent with observation or gedanken experiments?
Yes, absolutely. Clocks run slower, even light clocks, along with all physical processes. We have the GPS clock adjustment, and it's the Shapiro Delay. The time component of this is the most important, more important than any path change. 

Edit: Amrit is correct when he says this is not a new theory. The concept that "time is change" goes back to Aristotle, it resurfaced as Presentism in 1908, and was only fully appreciated by Godel and Einstein in 1949. And I'm afraid clocks don't measure time. What they actually measure is motion. This might be cyclical motion such as the turning of a cog or the vibration of a crystal or an electron bond, and we might label these cyclical motions as events. But the important thing to grasp is if that motion goes slower, the clock goes slower. It's not time going slower, it's motion. And that motion is through space. When you "stop the clock" or "freeze the frame" what you're actually stopping is motion. I've written an essay called "Time Explained" to flesh out the details of all this, but people consider this sort of thing to be "New Theories". That's because it's not in line with the common current concept of time as something that flows and that we travel through. It isn't, and we don't. There is absolutely no scientific evidence to back up this common concept of time, none whatsoever.
Title: Will a photon clock run at a different rate from an atomic clock under gravity?
Post by: graham.d on 25/05/2010 12:12:04
"Yes, both spaces are flat and local physical laws are the same. But whilst that  gravitational potential is relative, it does have an absolute meaning. It's saying something pretty fundamental about the nature of space at that location, and relative measurements tell you that the difference between locations is very real." - Farsight

This is what I was getting at but I have not really thought it through yet. Unfortunately I have to work too :-) If Gravitational Potential has an absolute range, then there must come a point where the this should be detectable within the spacetime region itself - like a BH singularity for example. It is unusual in nature to find brickwall phenomena where there is no effect of a parameter but when it reaches a certain value there is a dramatic change. Though, come to think of it, it may be that the assumption of flatness may start to be invalid, covering a smaller and smaller region, as the potential gets lower. Then this would be noticeable. Though this would depend on whether this changed faster than the the space compression.

The radial direction for the sqashing of space would presumably not be important if we could assume that the spacetime region was sufficiently flat (a small enough volume for example). I'm trying to stick with a Minkowski metric rather than a Schwartzchild metric - but maybe this cannot be done. Going back to the mirror spacing, this would mean the mirror spacing would be smaller as viwed by a distant observer at a higher potential. This would then mean that that c would have to change (be slower) with a higher order than the space compression to produce the gravitational time dilation. Have you a web reference that gives the coordinate tranformation that should be applied for a change in Gravitational Potential? It should take these effects into account.
Title: Will a photon clock run at a different rate from an atomic clock under gravity?
Post by: Farsight on 25/05/2010 15:17:44
There is a "brick wall" phenomenum, graham - at the event horizon of a black hole. The coordinate speed of light at the event horozon is zero according to all observers in the universe at large, and the radial length contraction is total. This is the Schwartzchild metric, where the singularity is at the event horizon "surface", rather than at a point in the centre.

No, I'm afraid I don't have a web reference for a coordinate tranformation that should be applied for a change in gravitational potential. This: http://casa.colorado.edu/~ajsh/schwp.html isn't bad, but the problem is that if light has stopped you don't actually have a coordinate system any more. All measurements take a finite time in your proper time, but forever as far as the rest of the universe is concerned, so those measurements are never actually made.

This is quite a good article too: http://www.mathpages.com/rr/s7-02/7-02.htm. The author doesn't agree with me, but he does mention the Weinberg field interpretation, which is what I'm talking about here.
Title: Will a photon clock run at a different rate from an atomic clock under gravity?
Post by: graham.d on 25/05/2010 15:45:14
Thanks, I'll take a look when time permits. I was referring to a coordinate transformation before the limit of a singularity although if it doesn't include that, it is incomplete. In any case, my point was to see whether the approach of the singularity was detectable (before the singularity was reached) from inside the spacetime region that was being "compressed". Having thought about it, I think not. I mentioned before that it may be increasingly difficult to have a region of approximately flat space approaching such a condition, and this would be detectable. This is analogous to increasing tidal forces when approaching a BH however, a sufficiently huge BH would allow crossing of such a horizon without noticeable tidal forces (i.e. local space is reasonably flat). To take this analogy further, does this mean the gravitational potential has gone negative? I appreciate this would have no meaning to the outside observer any more than the state of anything crossing a BH event horizon.

Mathpages are generally excellent - I don't know why I didn't look there anyway. I will take a look. Thanks.
Title: Will a photon clock run at a different rate from an atomic clock under gravity?
Post by: Geezer on 25/05/2010 17:28:11
Er, well the second is defined as the interval for a certain number of the atomic events. The number was chosen to line up with more traditional definitions for the second. As you point out, the fundamental "tick" is an electromagnetic event with the atom.
And if the space is different, the underlying motion associated with this event is slower. It's a spin-flip, the electron is turning over. You can get a feel for this by stretching an eleastic band over your finger and thumb. Stick a pencil in, and turn the pencil over 180 degrees tensioning the elastic, then let go. It vibrates back and forth.   

The light output could be delayed in a very variable fashion, but it would have no effect on the frequency of the microwave resonator. The light output is only sampled periodically to make very infrequent and very minute adjustments to the frequency of the microwave resonator. I believe the detector only responds to the number of photons that reach it. I'm not sure how they know to adjust up or down because I would assume that the light output will diminish if the resonator is slow or fast relative to the caesium events.
It makes no odds, electromagnetic phenomena propagate at a slower rate in a region of lower gravitational potential.

Being pedantic, you can't really say that the events are happening at a "slower rate", because that seems to suggest that time itself is invariant.
Things move slower, that's all. When the motion is cyclic like the spin-flip, we call the result an event.

LOL - it's not really "slower". If we are measuring time by counting atomic events, we always observe the same number of atomic events for a particular amount of time. My point is that we have to be very careful when we use measures that are a function of time.
Title: Will a photon clock run at a different rate from an atomic clock under gravity?
Post by: Geezer on 25/05/2010 18:41:59

I guess a good question is whether or not a "good" clock (meaning one that measures local time accurately) is in practice any different than measuring time itself.  I don't know the answer, but I've been assuming that it isn't.

JP - I think that any clock is actually measuring the passage of time in its locality. And clocks can include anything from atomic activity, chemical processes, large pendulums and rotating orbs. I think our intution suggests there is some sort of absolute time, even though science tells us it's not so.

We sense the the passage of time is smooth, but if it was in fact quite jittery I think it would be impossible to detect that it actually was jittery without some sort of absolute timing device which was independent of local time (which I think is impossible!)

I'm going to have to lie down for a bit now  [;D]
Title: Will a photon clock run at a different rate from an atomic clock under gravity?
Post by: amrit on 26/05/2010 08:02:23
Time does not run on its own. There is no passage of time.
Time dilatations means exactly that clock run slower.
Not because of time run slower.
There is no physical time behind run of a clock.
Clock run in a timeless space.
Title: Will a photon clock run at a different rate from an atomic clock under gravity?
Post by: Farsight on 26/05/2010 08:28:08
..In any case, my point was to see whether the approach of the singularity was detectable (before the singularity was reached) from inside the spacetime region that was being "compressed". Having thought about it, I think not.
You certainly won't observe the time dilation or the  length contraction locally, but there actually be a way to find out that things are changing by measuring the fine structure constant. Search the internet for "tests of Lorentz invariance" to read about other ideas for this. I'm all for relativity, but I wouldn't rule out some subtle breach here.

I mentioned before that it may be increasingly difficult to have a region of approximately flat space approaching such a condition, and this would be detectable. This is analogous to increasing tidal forces when approaching a BH however, a sufficiently huge BH would allow crossing of such a horizon without noticeable tidal forces (i.e. local space is reasonably flat). To take this analogy further, does this mean the gravitational potential has gone negative? I appreciate this would have no meaning to the outside observer any more than the state of anything crossing a BH event horizon.
I don't see how a gravitational potential can go negative. The gradient in gravitational potential is "why things fall down". If there's no gradient at all, things don't fall down. If the gradient is uniform you still fall down, but there's no tidal force. There's only a tidal force when the gradient changes say between your head and your feet. There's a gradient in the gradient, as it were.
Title: Will a photon clock run at a different rate from an atomic clock under gravity?
Post by: Farsight on 26/05/2010 08:39:49
LOL - it's not really "slower". If we are measuring time by counting atomic events, we always observe the same number of atomic events for a particular amount of time. My point is that we have to be very careful when we use measures that are a function of time.
It really is slower, Geezer. If you could actually film those atomic events at the different locations then put them up on a split screen, you'd see it. And the definition of the second is the other way round. The number of atomic events defines the second. It increases with reducing gravitational potential because the motion underlying the events is slower. Amrit is right. Time doesn't "run" and it doesn't "pass". When a clock goes slower it's because things move slower. Godel and Einstein really did work this out in 1949.
Title: Will a photon clock run at a different rate from an atomic clock under gravity?
Post by: Geezer on 26/05/2010 09:30:34
LOL - it's not really "slower". If we are measuring time by counting atomic events, we always observe the same number of atomic events for a particular amount of time. My point is that we have to be very careful when we use measures that are a function of time.
It really is slower, Geezer. If you could actually film those atomic events at the different locations then put them up on a split screen, you'd see it. And the definition of the second is the other way round. The number of atomic events defines the second. It increases with reducing gravitational potential because the motion underlying the events is slower. Amrit is right. Time doesn't "run" and it doesn't "pass". When a clock goes slower it's because things move slower. Godel and Einstein really did work this out in 1949.

OK - I'll admit I'm being pedantic, but I think it's important to precise when we talk about time.

I don't think we can really use the terms "slower" or "faster" because speed is a function of time, so there is no difference in speed. There is no "absolute" time, so when you say "slower", I think you really mean that the count of observed events at different locations can differ (assuming we have some method to observe or record those events.)
Title: Will a photon clock run at a different rate from an atomic clock under gravity?
Post by: graham.d on 26/05/2010 09:34:48
"I don't see how a gravitational potential can go negative. The gradient in gravitational potential is "why things fall down". If there's no gradient at all, things don't fall down. If the gradient is uniform you still fall down, but there's no tidal force. " - Farsight

I do understand this Farsight but linearity in potential, in itself, does not preclude a negative value. You can have linearity in Potential from + to - infinity with respect to an arbtrary spacial measure (say r) and its differential with respect to r would be a constant. It is also obvious that it would be the second derivative that would result in tidal forces.

My point was simply that someone can pass through a BH event horizon and there would be no specific local measurement to mark that event providing the BH were large enough to minimise destructive tidal forces. But what would be their Gravitational Potential? There is no doubt that it would continue to fall as they moved toward the BH centre. From a distant observer's perspective they would have zero GP as they reach the EH but that time dilation would infinitely delay the event. In this sense negative GP values are "cosmically censored". From the perspective of the person crossing a BH EH, I guess it would not be the first thing on his mind. I'm never sure what he would see though there are some interesting simulations and animations on the web. Perhaps he should see the distant observer having infinite GP.

Anyway, this is not so profound as I first thought and is not leading to any great insight. It does suggest that GP would depend on from whose perspective it is being measured and that the scale can slide so as to define the zero value at (say) an EH.
Title: Will a photon clock run at a different rate from an atomic clock under gravity?
Post by: amrit on 26/05/2010 11:34:14
LOL - it's not really "slower". If we are measuring time by counting atomic events, we always observe the same number of atomic events for a particular amount of time. My point is that we have to be very careful when we use measures that are a function of time.
It really is slower, Geezer. If you could actually film those atomic events at the different locations then put them up on a split screen, you'd see it. And the definition of the second is the other way round. The number of atomic events defines the second. It increases with reducing gravitational potential because the motion underlying the events is slower. Amrit is right. Time doesn't "run" and it doesn't "pass". When a clock goes slower it's because things move slower. Godel and Einstein really did work this out in 1949.

OK - I'll admit I'm being pedantic, but I think it's important to precise when we talk about time.

I don't think we can really use the terms "slower" or "faster" because speed is a function of time, so there is no difference in speed. There is no "absolute" time, so when you say "slower", I think you really mean that the count of observed events at different locations can differ (assuming we have some method to observe or record those events.)

speed is not function of time, speed is a function of motion that run in space (and not in time)
ve measure speed with clocks

v = d/tn  where tn is a last "tick" of clock.
Title: Will a photon clock run at a different rate from an atomic clock under gravity?
Post by: JP on 26/05/2010 11:46:47
speed is not function of time, speed is a function of motion that run in space (and not in time)
ve measure speed with clocks

v = d/tn  where tn is a last "tick" of clock.

I disagree.  Everything I've ever learned in mainstream physics on the subject says that speed a ratio of change in position to change in time.  Classically speaking, speed=dx/dt (dx distance moved in a short time, dt).  Do you have a source for your claim?
Title: Will a photon clock run at a different rate from an atomic clock under gravity?
Post by: amrit on 26/05/2010 12:53:33
speed is not function of time, speed is a function of motion that run in space (and not in time)
ve measure speed with clocks

v = d/tn  where tn is a last "tick" of clock.

I disagree.  Everything I've ever learned in mainstream physics on the subject says that speed a ratio of change in position to change in time.  Classically speaking, speed=dx/dt (dx distance moved in a short time, dt).  Do you have a source for your claim?

Yes now we will change this idea as there is no experimental data for it. Nothing happens in time as time is a mind frame through which we experience motion in space.
Title: Will a photon clock run at a different rate from an atomic clock under gravity?
Post by: Farsight on 26/05/2010 13:53:20
I disagree. Everything I've ever learned in mainstream physics on the subject says that speed [is] a ratio of change in position to change in time.
There no easy way to say this, JP, but it has to be said: everything you've ever learned on the subject is wrong. It sounds awful I know, but look to the evidence. What we actually see is space and motion through it. We don't see time flowing or any travel through time. The motion is through space. So motion gains precedence over time. Time is reduced to a cumulative measure of local motion, such as the motion of light or caesium spin-flips. For a reference, see A World Without Time: The Forgotten Legacy of Godel and Einstein (http://www.amazon.co.uk/World-Without-Time-Forgotten-Einstein/dp/0713993871). Godel and Einstein worked it out in 1949: you don't need time to have motion, you need motion to have time.

Amrit is right. Shame he's wrong about that photon clock, but hey-ho. Hi amrit!
Title: Will a photon clock run at a different rate from an atomic clock under gravity?
Post by: Geezer on 26/05/2010 18:45:32
speed is not function of time, speed is a function of motion that run in space (and not in time)
ve measure speed with clocks

v = d/tn  where tn is a last "tick" of clock.

I disagree.  Everything I've ever learned in mainstream physics on the subject says that speed a ratio of change in position to change in time.  Classically speaking, speed=dx/dt (dx distance moved in a short time, dt).  Do you have a source for your claim?

Yes now we will change this idea as there is no experimental data for it. Nothing happens in time as time is a mind frame through which we experience motion in space.

JP is providing the accepted definition for speed. If you remove time from the definition, I think you are referring to something other than speed. To avoid confusion you should probably use a different name for whatever it is you are referring to.

Mind you, it would be good if you are correct, because, if you are, it means I'm not really getting older. I just think I'm getting older as I move around in space  [;D]
Title: Will a photon clock run at a different rate from an atomic clock under gravity?
Post by: JP on 27/05/2010 02:45:15
I disagree. Everything I've ever learned in mainstream physics on the subject says that speed [is] a ratio of change in position to change in time.
There no easy way to say this, JP, but it has to be said: everything you've ever learned on the subject is wrong. It sounds awful I know, but look to the evidence. What we actually see is space and motion through it. We don't see time flowing or any travel through time. The motion is through space. So motion gains precedence over time. Time is reduced to a cumulative measure of local motion, such as the motion of light or caesium spin-flips. For a reference, see A World Without Time: The Forgotten Legacy of Godel and Einstein (http://www.amazon.co.uk/World-Without-Time-Forgotten-Einstein/dp/0713993871). Godel and Einstein worked it out in 1949: you don't need time to have motion, you need motion to have time.

Amrit is right. Shame he's wrong about that photon clock, but hey-ho. Hi amrit!

I did a bit of poking around to find out what Godel actually did, scientifically.  What Godel did, it seems, was to work out solutions to Einstein's field equations that show that an object moving normally through the universe (i.e. on time-like paths) could go back in time. 

Does this cause problems with physics in our universe?  Almost certainly not, since Godel's universes are not our universe.  It might have philosophical implications about time being a "special" dimension. 

At any rate, back to your post.  I disagree with the conclusions you're drawing.  We certainly don't see "space and motion through it."  What we "see" (and how GR describes the universe) is paths in space-time.  What we locally define as velocity is a measurement of the slope between the space and time components of that path at any point.  Since these paths are geometrical objects, what you can physically measure are distances.  From these distances you can derive slopes.  Therefore, time is a fundamental quantity (it's a measure of "distance") and speed is a derived quantity (it's a measure of slope, or a ratio of "distances.")

Can you provide any scientific evidence to the contrary?
Title: Will a photon clock run at a different rate from an atomic clock under gravity?
Post by: amrit on 27/05/2010 07:22:43
I disagree. Everything I've ever learned in mainstream physics on the subject says that speed [is] a ratio of change in position to change in time.
There no easy way to say this, JP, but it has to be said: everything you've ever learned on the subject is wrong. It sounds awful I know, but look to the evidence. What we actually see is space and motion through it. We don't see time flowing or any travel through time. The motion is through space. So motion gains precedence over time. Time is reduced to a cumulative measure of local motion, such as the motion of light or caesium spin-flips. For a reference, see A World Without Time: The Forgotten Legacy of Godel and Einstein (http://www.amazon.co.uk/World-Without-Time-Forgotten-Einstein/dp/0713993871). Godel and Einstein worked it out in 1949: you don't need time to have motion, you need motion to have time.

Amrit is right. Shame he's wrong about that photon clock, but hey-ho. Hi amrit!

Yes this picture here on the first page how mirrors are moving and so photon has longer way and photon clock ticks slower is ridiculous.
Mora than that: when you move mirrors that atom run between them along the direction of the fast aircraft motion that photon clock should shrink and so “tick” faster what is contrary to SR
Title: Will a photon clock run at a different rate from an atomic clock under gravity?
Post by: amrit on 27/05/2010 07:26:06
I disagree. Everything I've ever learned in mainstream physics on the subject says that speed [is] a ratio of change in position to change in time.
There no easy way to say this, JP, but it has to be said: everything you've ever learned on the subject is wrong. It sounds awful I know, but look to the evidence. What we actually see is space and motion through it. We don't see time flowing or any travel through time. The motion is through space. So motion gains precedence over time. Time is reduced to a cumulative measure of local motion, such as the motion of light or caesium spin-flips. For a reference, see A World Without Time: The Forgotten Legacy of Godel and Einstein (http://www.amazon.co.uk/World-Without-Time-Forgotten-Einstein/dp/0713993871). Godel and Einstein worked it out in 1949: you don't need time to have motion, you need motion to have time.

Amrit is right. Shame he's wrong about that photon clock, but hey-ho. Hi amrit!

I did a bit of poking around to find out what Godel actually did, scientifically.  What Godel did, it seems, was to work out solutions to Einstein's field equations that show that an object moving normally through the universe (i.e. on time-like paths) could go back in time. 

Does this cause problems with physics in our universe?  Almost certainly not, since Godel's universes are not our universe.  It might have philosophical implications about time being a "special" dimension. 

At any rate, back to your post.  I disagree with the conclusions you're drawing.  We certainly don't see "space and motion through it."  What we "see" (and how GR describes the universe) is paths in space-time.  What we locally define as velocity is a measurement of the slope between the space and time components of that path at any point.  Since these paths are geometrical objects, what you can physically measure are distances.  From these distances you can derive slopes.  Therefore, time is a fundamental quantity (it's a measure of "distance") and speed is a derived quantity (it's a measure of slope, or a ratio of "distances.")

Can you provide any scientific evidence to the contrary?

There is no time in the universe. Photon (and all other motions) moves in space only. With clocks we measure numerical order of motion.
Never until know time was measured or observed as motion is observed and experienced. We measure with clocks motion and not time. Time is in the mind, we experience motion in time "past-present-future" that is a mind creation. Universe is without time as predicted by Einstein and Godel. See my post "Block Universe" in new theories.
Title: Will a photon clock run at a different rate from an atomic clock under gravity?
Post by: JP on 27/05/2010 09:25:30
There is no time in the universe. Photon (and all other motions) moves in space only. With clocks we measure numerical order of motion.
You're contradicting yourself.  An order of events implies time.  We see things in a certain order because we move through time. If there was no time, all events would be simultaneous and we couldn't order them.

Quote
Time is in the mind, we experience motion in time "past-present-future" that is a mind creation.
That's a philosophical point more than a scientific one.  It also requires that time exists to be perceived.

Quote
Universe is without time as predicted by Einstein and Godel. See my post "Block Universe" in new theories.
I disagree, as do Einstein's (and presumably and Godel's equations).  Special and general relativity assume a time coordinate.  I think what Godel was getting at is that something could in some theoretical universes move through space without moving through time.  That's completely different from saying that time doesn't exist.  We are, after all, perfectly happy with moving through time without moving through space--and we don't therefore claim that space doesn't exist.

If I'm interpreting what they said wrong, please let me know.
Title: Will a photon clock run at a different rate from an atomic clock under gravity?
Post by: amrit on 27/05/2010 10:32:31
JP

X4 = i x c x t (1) where t is "tick" of clock in space. Clock tick in space.
(1) is equal to d = v x t
X4 is not temporal coordinate, X4 is spatial too.
We live in a 4D space.
Time is a mind model throuht we experience motion in 4D,
see our article on FQXI
http://www.fqxi.org/data/forum-attachments/Block_Universe.pdf

Title: Will a photon clock run at a different rate from an atomic clock under gravity?
Post by: Farsight on 27/05/2010 12:34:47
I do understand this Farsight but linearity in potential, in itself, does not preclude a negative value. You can have linearity in Potential from + to - infinity with respect to an arbtrary spacial measure (say r) and its differential with respect to r would be a constant. It is also obvious that it would be the second derivative that would result in tidal forces.
Sorry to be slow getting back to you graham. I beg to differ on the negative value. The coordinate speed of light when measured from afar reduces with gravitational potential, and there's no problem with this reducing to zero. But it just can't go negative. A negative speed has no physical reality. Any solutions based on the maths here are non-real solutions. It's like asserting the reality of a carpet measuring -4 metres x -4 metres because it has an area of 16 square metres.   

My point was simply that someone can pass through a BH event horizon and there would be no specific local measurement to mark that event providing the BH were large enough to minimise destructive tidal forces.
They can't pass through it. The coordinate speed of light when measured by you and me tends to zero at the EH. That means all motion just... stops. Do read up on that Weinberg field interpretation. Once you suss it you'll know what I mean. 

But what would be their Gravitational Potential? There is no doubt that it would continue to fall as they moved toward the BH centre. From a distant observer's perspective they would have zero GP as they reach the EH but that time dilation would infinitely delay the event.
That's just it. It never happens. Their continued fall only occurs in an abstract "never-never land" beyond the end of time.

In this sense negative GP values are "cosmically censored". From the perspective of the person crossing a BH EH, I guess it would not be the first thing on his mind. I'm never sure what he would see though there are some interesting simulations and animations on the web. Perhaps he should see the distant observer having infinite GP.
He doesn't see anything. Light has stopped. Coordinate transformations do not take adequate account of this.

Anyway, this is not so profound as I first thought and is not leading to any great insight. It does suggest that GP would depend on from whose perspective it is being measured and that the scale can slide so as to define the zero value at (say) an EH.
It's all very simple stuff. It's much more mundane than the mooted point-singularites, and there are no infinities, just a c=0 at the event horizon. 
Title: Will a photon clock run at a different rate from an atomic clock under gravity?
Post by: Farsight on 27/05/2010 13:13:12
I did a bit of poking around to find out what Godel actually did, scientifically.  What Godel did, it seems, was to work out solutions to Einstein's field equations that show that an object moving normally through the universe (i.e. on time-like paths) could go back in time. Does this cause problems with physics in our universe? Almost certainly not, since Godel's universes are not our universe. It might have philosophical implications about time being a "special" dimension.
But then Godel reasoned that "if you could visit the past, time cannot have passed". He used this to show the impossibility of time travel, not the possibility. Search A World Without Time: The Forgotten Legacy of Godel and Einstein (http://www.amazon.co.uk/World-Without-Time-Forgotten-Einstein/dp/0713993871) on "time cannot have passed" and look at pages 129 and 130. What you read when you poke around is a corrupted conclusion, rather like the Schrodinger's cat example.  

At any rate, back to your post.  I disagree with the conclusions you're drawing. We certainly don't see "space and motion through it."  What we "see" (and how GR describes the universe) is paths in space-time.
We absolutely do not. We can't see a world-line, not at all. It's an abstract concept that is useful in calculations, but it isn't something you can actually observe.

What we locally define as velocity is a measurement of the slope between the space and time components of that path at any point.
The path isn't something real and observable. What's real and observable is an object moving through space. You can observe it because light moves from the object to your eye, and because signals are moving around in your brain.

Since these paths are geometrical objects...
But they're abstract geometrical objects. You can't actually see them. Nor can you point up to the night sky and say look, there's a light cone, or look, there's a reference frame. The important point to all this is to set aside the mathematical abstractions one is accustomed to working with, and look very hard at what you actually can see. Look inside a clock, and you see things moving, not time flowing. What's out there is space and motion through it. Things travel through space, they don't actually travel through time. People say things like "we travel forward in time at one second per second", but we don't travel at all. Time travel is science fiction. Ever heard of a stasis box? That's science-fiction too, but it's useful to demonstrate something: get in the box, and the "stasis field" prevents all motion, even at the atomic level. So you can't move, your heart doesn't beat, and you can't even think. When I open the box five hundred years later, to you it's like I opened the box as soon as you got in. You "travelled" to the future by not moving at all. Instead everything else did. And all that motion, be it the motion of planets or people or atoms or light, is through space.

...what you can physically measure are distances.
Yes, you can measure distances, but look at how you do it. The metre is defined as the distance travelled by light in a complete vacuum in 1⁄299,792,458 of a second. You're using the motion of light to define your distance.

From these distances you can derive slopes. Therefore, time is a fundamental quantity (it's a measure of "distance") and speed is a derived quantity (it's a measure of slope, or a ratio of "distances.")
I'm afraid that's incorrect. You define both the second and the metre using the motion of light, and that means motion is the fundamental quantity, along with space. Time is the emergent property, not motion. 

Can you provide any scientific evidence to the contrary?
Yes, the definition of the second and the metre, along with pair production, the Shapiro delay, the GPS clock adjustment, and of course the NIST fountain clock. Geezer objects to what I said about the latter, but pair production does tell us that the electron is literally made from light, so it's reasonable to assert that the electron spin-flip hyperfine transition in a Caesium atom is some kind of electromagnetic change or rotation or translation or motion. When the motion is slower in a region of low gravitational potential, the second is bigger, and we call it gravitational time dilation. You might have some difficulty accepting this, but there's absolutely no scientific evidence for time is running slower. None whatsoever.
Title: Will a photon clock run at a different rate from an atomic clock under gravity?
Post by: Farsight on 27/05/2010 13:37:58
...An order of events implies time. We see things in a certain order because we move through time. If there was no time, all events would be simultaneous and we couldn't order them.
Apologies amrit, but if I can contribute here: we see things in a certain order because motion is ordered from A to B to C, and that motion is through space. We don't "move through time", that's just a figure of speech. Those events are separated by intervening motion which in turn can be construed as other events. We call it time, and time does "exist". But not the way you think. It exists like heat exists, being an emergent property of motion. If you think of the kinetic theory of gases, you can see that temperature is a measure of average motion, whilst heat capacity and specific heat take in the characteristics of the subject. Time is a cumulative measure of motion, the baseline being the motion of light. The universe has been going for 13.7 billion light years. And like amrit says, clocks clock up motion. You can't open the back of a clock and see time flowing or any motion through time. Just motion, through space. 

Quote from: amrit
Time is in the mind, we experience motion in time "past-present-future" that is a mind creation.
That's a philosophical point more than a scientific one. It also requires that time exists to be perceived.
It is a scientific point, JP. Show me the scientific evidence for "we move through time". When you can't, you'll understand why. I'd go so far as to say that there is no scientific point that is more important than this one.

I disagree, as do Einstein's (and presumably and Godel's equations). Special and general relativity assume a time coordinate.
There's still a time coordinate, but it's a coordinate in a derived dimension. It's derived from motion through space. It isn't something that offers freedom of motion. You can move through space by hopping a metre backwards or forwards. You can't do this with time. 

I think what Godel was getting at is that something could in some theoretical universes move through space without moving through time.  That's completely different from saying that time doesn't exist.  We are, after all, perfectly happy with moving through time without moving through space - and we don't therefore claim that space doesn't exist.
See above.

If I'm interpreting what they said wrong, please let me know.
I'm afraid you are. People take this different viewpoint to mean "time does not exist". It isn't like that. It just isn't what you think it is. 
Title: Will a photon clock run at a different rate from an atomic clock under gravity?
Post by: JP on 27/05/2010 14:12:29
It is a scientific point, JP. Show me the scientific evidence for "we move through time". When you can't, you'll understand why. I'd go so far as to say that there is no scientific point that is more important than this one.

The space-time interval is:

ds2=dx2-c2dt2.

If dt2>0, you've moved through time.  For light, this interval is zero, so it's required to move through time. 

If you're claiming time is a derived dimension, can you provide evidence of it?  How would one revise relativity theory to account for this fact?
Title: Will a photon clock run at a different rate from an atomic clock under gravity?
Post by: Farsight on 27/05/2010 15:00:56
If you're claiming time is a derived dimension, can you provide evidence of it?
Yes. Observe a motionless object. It isn't moving. We derive the time dimension and the concept that the object is "moving through time" because everything else is moving, including light, brain signals, etc. Recognising this is seeing "what's actually there" rather than abstract things that aren't actually there. It can be difficult to do without a rigourous adherence to what can I see?, which takes considerable practice. 

How would one revise relativity theory to account for this fact?
In a rather minor ontological way that takes it back to the original. Einstein's The Foundation of the General Theory of Relativity gave the equations of motion, not the equations of curved spacetime. We still employ Minkowski spacetime, but we put the emphasis on motion and recognise that worldlines are plots rather than something we can observe or that an object actually moves along. The interpretation of the mathematics changes rather than the mathematics itself. Despite the fairly minor change, this old "true-to-Einstein" interpretation isn't considered to be mainstream, and tends to be discounted on those grounds alone rather than on grounds of empirical observable scientific evidence.
Title: Will a photon clock run at a different rate from an atomic clock under gravity?
Post by: Geezer on 27/05/2010 19:27:52
Yes, the definition of the second and the metre, along with pair production, the Shapiro delay, the GPS clock adjustment, and of course the NIST fountain clock. Geezer objects to what I said about the latter, but pair production does tell us that the electron is literally made from light, so it's reasonable to assert that the electron spin-flip hyperfine transition in a Caesium atom is some kind of electromagnetic change or rotation or translation or motion. When the motion is slower in a region of low gravitational potential, the second is bigger, and we call it gravitational time dilation. You might have some difficulty accepting this, but there's absolutely no scientific evidence for time is running slower. None whatsoever.

Farsight, my objections were mainly to your description of the operation of the NIST clock and I did agree with you that the fundamental action in the caesium atoms is electromagnetic. I think I also pointed out that clocks simply count events and that we can observe time dilation because of differences in the count of events.

As you point out, we measure the thing we refer to as time by using clocks to count events. We are able to observe that the effects of motion and gravity produce different event counts. Therefore, by definition, we observe relative differences in time i.e., relative fastness or slowness. (Also, please bear in mind that very many forms of matter can be used as "clocks" in one way or another.)

To support your position that there is no relative speed difference (and therefore time as we know it does not exist) I think you will have to produce an alternative definition for whatever it is you maintain is the thing we currently know as time.

If you can't provide a definition that other people can make use of to some advantage, it's likely that everyone will simply stick with the current definition of "time" because it seems to work quite well for their purposes.

Title: Will a photon clock run at a different rate from an atomic clock under gravity?
Post by: amrit on 27/05/2010 21:57:52
It is a scientific point, JP. Show me the scientific evidence for "we move through time". When you can't, you'll understand why. I'd go so far as to say that there is no scientific point that is more important than this one.

The space-time interval is:

ds2=dx2-c2dt2.

If dt2>0, you've moved through time.  For light, this interval is zero, so it's required to move through time. 

If you're claiming time is a derived dimension, can you provide evidence of it?  How would one revise relativity theory to account for this fact?

"cdt" is distance as "v x t" is distance. t in this formula is "tick" of clock running in space that itself is timeless
Title: Will a photon clock run at a different rate from an atomic clock under gravity?
Post by: amrit on 27/05/2010 22:01:37
...An order of events implies time. We see things in a certain order because we move through time. If there was no time, all events would be simultaneous and we couldn't order them.
Apologies amrit, but if I can contribute here: we see things in a certain order because motion is ordered from A to B to C, and that motion is through space. We don't "move through time", that's just a figure of speech. Those events are separated by intervening motion which in turn can be construed as other events. We call it time, and time does "exist". But not the way you think. It exists like heat exists, being an emergent property of motion. If you think of the kinetic theory of gases, you can see that temperature is a measure of average motion, whilst heat capacity and specific heat take in the characteristics of the subject. Time is a cumulative measure of motion, the baseline being the motion of light. The universe has been going for 13.7 billion light years. And like amrit says, clocks clock up motion. You can't open the back of a clock and see time flowing or any motion through time. Just motion, through space. 

Quote from: amrit
Time is in the mind, we experience motion in time "past-present-future" that is a mind creation.
That's a philosophical point more than a scientific one. It also requires that time exists to be perceived.
It is a scientific point, JP. Show me the scientific evidence for "we move through time". When you can't, you'll understand why. I'd go so far as to say that there is no scientific point that is more important than this one.

I disagree, as do Einstein's (and presumably and Godel's equations). Special and general relativity assume a time coordinate.
There's still a time coordinate, but it's a coordinate in a derived dimension. It's derived from motion through space. It isn't something that offers freedom of motion. You can move through space by hopping a metre backwards or forwards. You can't do this with time. 

I think what Godel was getting at is that something could in some theoretical universes move through space without moving through time.  That's completely different from saying that time doesn't exist.  We are, after all, perfectly happy with moving through time without moving through space - and we don't therefore claim that space doesn't exist.
See above.

If I'm interpreting what they said wrong, please let me know.
I'm afraid you are. People take this different viewpoint to mean "time does not exist". It isn't like that. It just isn't what you think it is. 

Farsight, time is a mind frame through which we experience motion in timeless space.
Title: Will a photon clock run at a different rate from an atomic clock under gravity?
Post by: chocochoco on 28/05/2010 05:04:06
Woah. It's so interesting to me.
Title: Will a photon clock run at a different rate from an atomic clock under gravity?
Post by: amrit on 28/05/2010 07:26:45
Woah. It's so interesting to me.

great, read ma full paper Block Universe
http://vixra.org/abs/1005.0098
Title: Will a photon clock run at a different rate from an atomic clock under gravity?
Post by: Farsight on 28/05/2010 12:47:59
Farsight, my objections were mainly to your description of the operation of the NIST clock and I did agree with you that the fundamental action in the caesium atoms is electromagnetic. I think I also pointed out that clocks simply count events and that we can observe time dilation because of differences in the count of events.
Sounds good to me, Geezer.

As you point out, we measure the thing we refer to as time by using clocks to count events. We are able to observe that the effects of motion and gravity produce different event counts. Therefore, by definition, we observe relative differences in time i.e., relative fastness or slowness. (Also, please bear in mind that very many forms of matter can be used as "clocks" in one way or another.)
Again, sounds good.

To support your position that there is no relative speed difference (and therefore time as we know it does not exist) I think you will have to produce an alternative definition for whatever it is you maintain is the thing we currently know as time.
There must be some misunderstanding here. I've said there is a relative speed difference, challenging amrit's OP assertion. I've also said time exists like heat exists. It's just that it isn't something that really flows, and we don't really travel through it. 

If you can't provide a definition that other people can make use of to some advantage, it's likely that everyone will simply stick with the current definition of "time" because it seems to work quite well for their purposes.
This is the precis:

Time exists like heat exists, being an emergent property of motion. It's a cumulative measure of motion used in the relative measure of motion compared to the motion of light, and the only motion is through space. So time has no length, time doesn’t flow and we don’t travel through it.

Quote from: amrit
Farsight, time is a mind frame through which we experience motion in timeless space.
I know what you mean, amrit. I use somewhat different language, but the underlying meaning is the same - what we actually observe is space and motion through it. And yet it is so very difficult to get people to let go of the idea that time flows and we travel through it, despite the total lack of scientific evidence to support such concepts.
Title: Will a photon clock run at a different rate from an atomic clock under gravity?
Post by: PhysBang on 28/05/2010 15:30:12
In a rather minor ontological way that takes it back to the original. Einstein's The Foundation of the General Theory of Relativity gave the equations of motion, not the equations of curved spacetime.
Could you please give an example of this? Can you at least give an example of Einstein giving equations of motion without a time coordinate?

Well, OK , I know that it is impossible to give an example of these because your claims are simply not true. Special and General Relativity are both fundamentally about how we can set up our coordinates however we want, up to certain limits, but they always include spacial coordinates and a time coordinate.
Title: Will a photon clock run at a different rate from an atomic clock under gravity?
Post by: Farsight on 28/05/2010 17:11:22
In a rather minor ontological way that takes it back to the original. Einstein's The Foundation of the General Theory of Relativity gave the equations of motion, not the equations of curved spacetime.
Could you please give an example of this? Can you at least give an example of Einstein giving equations of motion without a time coordinate?
No. The time coordinate remains. All that changes is the underlying meaning of what it represents.

Well, OK , I know that it is impossible to give an example of these because your claims are simply not true...
They are true. What isn't true is that time flows or that we travel through it. There's no scientific evidence whatsoever for time flowing or for time travel. These things are science fiction, not science.     

Special and General Relativity are both fundamentally about how we can set up our coordinates however we want, up to certain limits, but they always include spatial coordinates and a time coordinate.
And that time coordinate designates a "position" in a "dimension" that is merely a measure of cumulative spatial motion calibrated against the motion of light. This position is not a real position, and the time dimension isn't a dimension like the dimensions of space. It offers no freedom of motion. There is no motion through it, and no flow, and no travel. Now go and read Time Explained and understand it. It's very simple. All you have to do is look at the observational evidence that's there in front of you, and admit to yourself that a clock does what you can see it doing. A clock "clocks up" motion, not time. 

Sorry amrit, I didn't mean to hijack your thread. I'll take a back seat. 
Title: Will a photon clock run at a different rate from an atomic clock under gravity?
Post by: amrit on 28/05/2010 20:26:18
"Time exists like heat exists, being an emergent property of motion"...............Farsight tell me how time is property of motion, explain in deatails.
Title: Will a photon clock run at a different rate from an atomic clock under gravity?
Post by: Farsight on 29/05/2010 15:06:42
TIME EXPLAINED

Time is very simple, once you get it. But “getting it” is so very difficult. That’s because your current concept of time is so deeply ingrained. You form a mental map of the world using your senses and your brain. You use this mental map to think, and you are so immersed in it that you can’t see things the way they really are. You are locked into an irrational conviction that clocks run, that days pass, that time flows, and that a journey takes a length of time. It takes steely logic to break out of this conditioning. First of all we need to look at your senses and the things you experience. Let’s start with sight. Look at the picture below:
 
(https://www.thenakedscientists.com/forum/proxy.php?request=http%3A%2F%2Fphotos1.blogger.com%2Fblogger%2F1491%2F82%2F320%2Fcheckerboardillusion.jpg&hash=6bc7c13da8e0ff440c9fa7427133a01c)

Now, squares A and B are the same colour. They’re the same shade of grey. Oh no they’re not, I hear you say. Oh yes they are I insist. Oh no they’re not you answer back. We could do this all day, but they really are the same colour. Squares A and B are the same shade of grey. The apparent difference in colour is an illusion. Just look at the screen from a narrow angle to break the illusion. See  http://web.mit.edu/persci/people/adelson/checkershadow_illusion.html for more details. Check it out for yourself. Satisfy yourself. Be empirical, test yourself, then you realise that A and B really are the same colour.

What this tells you is that colour is subjective. It isn’t a real property of things in the world. It’s perception, a "quale", and it’s in your head. A photon doesn’t actually have a colour. It has a wavelength, an oscillation, a frequency. What’s it’s got is a motion.

Let’s move on to sound. Imagine a super-evolved alien bat with a large number of ears, like a fly’s eye. This bat would “see” using sound, and if it was sufficiently advanced it might even see in colour. But we know that sound is pressure waves, and when we look beyond this at the air molecules, we know that sound relies on motion.

(https://www.thenakedscientists.com/forum/proxy.php?request=http%3A%2F%2Ft1.gstatic.com%2Fimages%3Fq%3Dtbn%3A_KM9q4067LihiM%3Ahttp%3A%2F%2Fmatthewpippin.com%2Fimages%2FE_Images%2FEducation_LWW%2FBatSonarOriginalArt7LWW.jpg&hash=bfd9def69d4a8af2ae9d24a60f21ba17)

Pressure is related to sound, and to touch. You feel it in your ears on a plane, or on your chest if you dive. This pressure of air or water is not some property of the sub-atomic world. It’s a derived effect, and the Kinetic Theory of Gases tells us that pressure is derived from motion.

You can also feel kinetic energy. If a cannonball in space travelling at 1000m/s impacted your chest you would feel it for sure. But apologies, my mistake. It isn't the cannonball doing 1000m/s. It's you. So where's the kinetic energy now? Can you feel it coursing through your veins? No. Because what’s really there is mass, and relative motion.

You can also feel heat. Touch that stove and you feel that heat. We talk about heat exchangers and heat flow as if there’s some magical mysterious fluid in there. And yet we know there isn’t. We know that heat is another derived effect of motion.

(https://www.thenakedscientists.com/forum/proxy.php?request=http%3A%2F%2Fwww.uwsp.edu%2Fphysastr%2Fkmenning%2Fimages%2Fmolecules.gif&hash=17e1b7a0e8b6259205098d811bc58e85)

Taste is chemical in nature, and somewhat primitive. Most of your sense of taste is in fact your sense of smell. Do you know how smell works? Look up olfaction and you’ll learn about molecular shape. But the latest theory from a man called Luca Turin says it’s all down to molecular vibration, because isomers smell the same. That’s motion again.

The point of all this is there’s a lot of motion out there, and most of your senses are motion detectors. But it probably never occurred to you because you’re accustomed to thinking about the world in terms of how you experience it, rather than the scientific, empirical, fundamental, ontological things that are there.

And nowhere is this more so than with time.

So, what is time? Let’s start by looking up the definition of a second:

Under the International System of Units, the second is currently defined as the duration of 9,192,631,770 periods of the radiation corresponding to the transition between the two hyperfine levels of the ground state of the caesium-133 atom...

So, a second is nine billion periods of radiation. But what is a period? We know that radiation is electromagnetic in nature, the thing we commonly call light. We also know that light has a frequency. So let’s look at frequency:

Frequency = 1 / T and Frequency = v / λ

This says frequency is the reciprocal of the period T, and is also velocity v divided by wavelength λ . Combining the two, we can say T = λ / v, which means a period T is a wavelength λ divided by a velocity v. To try to find out more, we can drill down into wavelength and velocity. We know that a wavelength is a distance, a thing like a metre:

The metre is the length of the path travelled by light in vacuum during a time interval of 1/299792458 of a second...

And we know already that a velocity is a distance divided by a time. So if a period is a wavelength divided by a velocity, that means a period is a distance divided by a distance divided by a time. So let’s do some simple mathematics. Let’s work it through. We can combine T = λ / v and v = λ / t and write it down as:

T = λ / ( λ / t)

Then we can cancel out the λs to get:

T = 1/(1/t)

Then we cancel the double reciprocal to leave:

T = t

The answer we get is T = t. A period of time is a period of time. This mathematical definition of time is circular. The mathematics tells us nothing about its base terms. So what is its true nature? How do we dig down and get to the bottom of it?

Let’s look at frequency some more. What’s the definition in English?

Frequency is the measurement of the number of times that a repeated event occurs per unit of time.

Our unit of time is the second. Frequency is the number of events per second. A second is nine billion periods of electromagnetic radiation. A period of radiation is an electromagnetic event, caused by an electromagnetic event happening inside an atom. For an event to happen, something has to move. Some component of the caesium atom has to travel some distance. A hyperfine transition is to do with magnetic dipole movement, a flip-flop interaction between the nucleus and an electron. It’s magnetic, so it’s electromagnetic in nature. Like the electron is electromagnetic in nature. Like the photon is electromagnetic in nature. So in some simple respect, we can consider some vital component of the atom to be electromagnetic just like light.
 
(https://www.thenakedscientists.com/forum/proxy.php?request=http%3A%2F%2Fwww.astro.rug.nl%2F%7Eweygaert%2FInleidingStk2%2Fimages%2Fspinflip.FG18_016.jpg&hash=914052ffc97e941de439f33e0147e8f9)

The answer comes with a rush. It’s a form of light moving inside the atom, and it causes more light, radiation with a frequency, waves with peaks, We sit there counting them as they go by, and when we get to nine billion, we say its a second. Then we use this second to measure the speed of light. We measure the speed of light in terms of the speed of light. In caesium atoms, in hydrogen atoms, in our own atoms, in the atoms of everything. No wonder it never changes.

And so the penny drops: the interval between events is measured in terms of other events. And the interval between those events is measured in terms of other events. Until there are no events left, only intervals. And intervals are frozen timeless moments. But you need events, not frozen timeless intervals to mark out the time. The events aren’t in the time, the time is in the events. Because time is merely the measure of events, of change, measured against some other change. And for things to change, there has to be motion. You don’t need time to have motion. You need motion to have time.

You don’t need regular atomic motion to mark out time. Any regular motion will do. Yes, we counted nine billion oscillations and called it second. One, two, three… nine billion. But you don’t have to count microwave wavepeaks emitted by a caesium atom. You could count beans in a bucket. Ping, ping, ping, chuck them in, regular as clockwork.

(https://www.thenakedscientists.com/forum/proxy.php?request=http%3A%2F%2Fhighcountrycoffee.com%2Fecommerce%2Fos%2Fcatalog%2Fimages%2Fbucket-of-beans.gif&hash=af1f62025fac9caa994a678704a4f83e)

You’re sitting there counting beans into the bucket, ping, ping, ping, regular as clockwork. Now, what is the direction of time? The only direction that is actually there, is the direction of the beans you’re throwing, and that direction is the direction of motion through space. A fuller bucket is not the direction of time. More beans is not the direction of time. The direction of time is the direction of your counting, and I could have asked you to count the beans out of the bucket. There is no real direction. It’s as imaginary as the direction you take when you count along the set of integers.

1 2 3 4 5 6 7 8 9 10 11 12 13 14 15 16 17 18 19 20 21 22 23 24 25 26 27 28 29 →

It’s imaginary, so you cannot actually point in this direction. Nor can an arrow. There is no Arrow of Beans, so there is no Arrow of Time. And since there’s no direction, there’s no direction you can possibly travel in. And since you can’t travel, you can’t travel a length, and a length can’t pass you by. It’s all abstraction, a false concept rooted in the language we use to think. Yet we never ever think about what the words actually mean. Instead we say the clock is running slow as if a clock is an athlete. We say the day went quickly but it didn’t go anywhere. We say years pass, but they don’t go by like buses.

(https://www.thenakedscientists.com/forum/proxy.php?request=http%3A%2F%2Ft0.gstatic.com%2Fimages%3Fq%3Dtbn%3AKYaAzWCvb76nWM%3Ahttp%3A%2F%2Fallstatepropertyonline.com%2Ffiles%2FRunningClock.gif&hash=7b5c9742c335008b863142d84b9a9e1f)

The only directions that are there, are the directions of the spatial motions that make the events that we use to measure the intervals between the other events. What’s there is the motion of light, the motion of atoms, and the motion of clocks, buses, and rivers. What’s there is the motion of the earth, and the sun, moon, and stars. And these motions are being counted, incremented, added up. We count regular atomic motion to use as a ratio against some other motion, be it of light, clocks, or buses. All of these things have motion, both internal motion and travelling motion. And all those motions are real, with real directions in space. But the time direction isn't real. It's as imaginary as a trip to nine billion.

That's why the past is only in your head, in your memory, in your records. It isn’t a place you can travel to. It’s just the places where things were. All those places that are still here in the universe. And while the past is the sum of all nows, now lasts for no time at all. Because there’s no time like the present, and time needs events, and when you take away the events, you take away the time. A second isn’t some slice of spacetime. It’s just nine billion motions of light from a caesium atom. Accelerate to half the speed of light and a second is still nine billion motions of light from a caesium atom. But there's only half the local motion there used to be, because the other half is already doing the travelling motion through space. That’s why time dilates.

It’s easy to understand time dilation. Imagine yourself as a metronome. Each tick is a thought in your head, a beat in your heart, a second of your time. If you’re motionless with respect to me I see you ticking like this: |||. If you flash by in a spaceship, I see you ticking like this: /\/\/\. If you could reach c and we know you can’t, you wouldn’t tick at all. Your time would flatline like this ______ because any transverse motion would cause c to be exceeded. You wouldn’t tick for me, you wouldn’t tick for you, and you wouldn’t tick for anybody else in the universe.

That’s the thing we’re interested in. The universe. That’s the thing that’s out there, the thing we’re a part of, the thing we’re trying to understand. It’s full of motion, and this is what it’s like:

(https://www.thenakedscientists.com/forum/proxy.php?request=http%3A%2F%2Fupload.wikimedia.org%2Fwikipedia%2Fcommons%2F6%2F6d%2FTranslational_motion.gif&hash=0657f9abf0cb8d4e595a08cda44fe9b8)

What can you see? What can you measure? You can measure the height. You can measure the width. And if it wasn't just a picture you could measure the depth. That's three Dimensions, with a capital D because we have freedom of movement in those dimensions. What else can you see? What else can you measure? You might imagine a fourth dimension, a time dimension. But the picture comes from the wikipedia temperature page. It’s a gif, a moving image, and in that image, those red and blue dots are moving. The thing you can measure is temperature.

Temperature is an aspect of heat, an emergent property, a derived effect of atomic and molecular motion. When you measure the temperature, you are measuring an aggregate motion. If you were one of those dots, you would not talk of climbing to a “higher temperature”. There is no real height. You can’t literally climb to a higher temperature. Hence we don’t call temperature a dimension. But people did. Temperature used to be called a dimension, but the word has gradually changed from its original meaning of “measure”, and is now assumed to be something that offers a degree of freedom, something you can move through.

We are immersed in time like the dots are immersed in temperature. It’s a different measure, but just as we cannot travel in temperature because there is no real height, we cannot travel in time because there is no real length. Because time is a dimension with a small d. There is no degree of freedom. I can hop backwards a metre but not backwards a second. Because time is a measure of change rather than a measure of place, and it has no absolute units, because you can only measure one change of place against another. It’s a relative measure of motion. The units are relative, and that’s what Special Relativity was telling us all along.

Special Relativity tells us that your relative velocity alters your measurement of space and time compared to everybody else. You increase your relative velocity and space appears to contract while time dilates by a factor of √(1-v²/c²). If you travel at .99c, space appears to contract to one seventh of its former size. So your trip to a star seven light years away only takes you a year. But physics is about the universe, and in that universe it took you seven years. The star didn’t become a disc because you flashed by. The space in the universe didn’t really contract because you travelled through it. But your time did.

(https://www.thenakedscientists.com/forum/proxy.php?request=http%3A%2F%2Fwww.thebigview.com%2Fspacetime%2Ftdgraphformula.gif&hash=b39fbe077f4e339f2802fec1adc69067)

Einstein didn’t quite understand the full meaning of relativity until later in life. He started off by saying there is no absolute time, using a postulate that says the the speed of light is always measured to be the same. But when he did general relativity, he said the speed of light varies with position. It it wasn’t until he was with Godel in Princeton that he really got it:
 
"It is a widely known but insufficiently appreciated fact that Albert Einstein and Kurt Godel were best friends for the last decade and a half of Einstein's life. They walked home together from Princeton's Institute for Advanced Study every day; they shared ideas about physics, philosophy, politics, and the lost world of German-Austrian science in which they had grown up. What is not widely known is that in 1949 Godel made a remarkable discovery: there exist possible worlds described by the theory of relativity in which time, as we ordinarily understand it, does not exist. He added a philosophical argument that demonstrates, by Godel's lights, that as a consequence, time does not exist in our world either. If Godel is right, Einstein has not just explained time; he has explained it away... (Palle Yourgrau, A World Without Time)".

(https://www.thenakedscientists.com/forum/proxy.php?request=http%3A%2F%2Fedge.org%2Fimages%2FEinstein.Godel.550.jpg&hash=19b6943b4f2e8e357816cc418e011c7d)

And what he got was this: time exists like heat exists. It’s real because it does things to us. But just like heat it’s an emergent property, a derived effect of motion. It means time is not fundamental. It isn’t a dimension like the dimensions of space. We don’t see four dimensions. We see space and motion through it. The thing called c is a conversion factor, between the measure of distance and the measure we call time, and both are derived from the motion of light. It’s the motion that’s king, the universe is not a block universe, it is a world in motion. The worldlines are only in mathematical space, and in your head. There’s no place that’s the future, and no place that’s the past. There’s only this place, and the time is always now. We don’t travel in time at one second per second. We don't travel in time at all. To travel backwards in time we'd need to unevent events, we’d need negative motion. But motion is motion whichever way it goes. You can’t have negative motion, just as you can’t have negative distance. Just as you can’t have negative carpets. So you can’t travel in time. There are no time travel paradoxes, because there is no time travel, and there is no time travel because time is just a relative measure of motion. And motion is travel. You can’t travel through travel.

So those celebrity physicists who talk earnestly of time machines are wrong. Dead wrong. Not even wrong. And all those folk who puzzle about the beginning of time are chasing the wrong horse. There never was any beginning of time. Time didn’t start thirteen point seven billion years ago. Because time didn’t start in the first place. It was motion that started in the first place. It was a place, not a time. And it’s this place, the place we call the universe, marked out by every light path you can track through timeless space. That’s how far we’ve come. A long long way, in no time at all
Title: Will a photon clock run at a different rate from an atomic clock under gravity?
Post by: amrit on 29/05/2010 21:44:28
Amrit, (to quote from Monty Python) this is contradiction, not argument.

"Space is timeless" is meaningless unless you explain your definitions
"'Velocity' of clocks" is also not what you mean (I think).
It has everything to do with the observer and the different gravitational potential. If you were to do the maths rigorously you would find the "spacetime interval" will be the same to all observers.
The rules of the universe are what they are and not what you choose them to be, so by all means do your experiment, but you seem to have presupposed the result. But if it turned out you were right you will surprise a lot of people :-)

space is timeless means that time is not part of the space, physical time is "tick" of clocks
Title: Will a photon clock run at a different rate from an atomic clock under gravity?
Post by: amrit on 29/05/2010 21:51:25
TIME EXPLAINED

Time is very simple, once you get it. But “getting it” is so very difficult. That’s because your current concept of time is so deeply ingrained. You form a mental map of the world using your senses and your brain. You use this mental map to think, and you are so immersed in it that you can’t see things the way they really are. You are locked into an irrational conviction that clocks run, that days pass, that time flows, and that a journey takes a length of time. It takes steely logic to break out of this conditioning. First of all we need to look at your senses and the things you experience. Let’s start with sight. Look at the picture below:
 
(https://www.thenakedscientists.com/forum/proxy.php?request=http%3A%2F%2Fphotos1.blogger.com%2Fblogger%2F1491%2F82%2F320%2Fcheckerboardillusion.jpg&hash=6bc7c13da8e0ff440c9fa7427133a01c)

Now, squares A and B are the same colour. They’re the same shade of grey. Oh no they’re not, I hear you say. Oh yes they are I insist. Oh no they’re not you answer back. We could do this all day, but they really are the same colour. Squares A and B are the same shade of grey. The apparent difference in colour is an illusion. Just look at the screen from a narrow angle to break the illusion. See  http://web.mit.edu/persci/people/adelson/checkershadow_illusion.html for more details. Check it out for yourself. Satisfy yourself. Be empirical, test yourself, then you realise that A and B really are the same colour.

What this tells you is that colour is subjective. It isn’t a real property of things in the world. It’s perception, a "quale", and it’s in your head. A photon doesn’t actually have a colour. It has a wavelength, an oscillation, a frequency. What’s it’s got is a motion.

Let’s move on to sound. Imagine a super-evolved alien bat with a large number of ears, like a fly’s eye. This bat would “see” using sound, and if it was sufficiently advanced it might even see in colour. But we know that sound is pressure waves, and when we look beyond this at the air molecules, we know that sound relies on motion.

(https://www.thenakedscientists.com/forum/proxy.php?request=http%3A%2F%2Ft1.gstatic.com%2Fimages%3Fq%3Dtbn%3A_KM9q4067LihiM%3Ahttp%3A%2F%2Fmatthewpippin.com%2Fimages%2FE_Images%2FEducation_LWW%2FBatSonarOriginalArt7LWW.jpg&hash=bfd9def69d4a8af2ae9d24a60f21ba17)

Pressure is related to sound, and to touch. You feel it in your ears on a plane, or on your chest if you dive. This pressure of air or water is not some property of the sub-atomic world. It’s a derived effect, and the Kinetic Theory of Gases tells us that pressure is derived from motion.

You can also feel kinetic energy. If a cannonball in space travelling at 1000m/s impacted your chest you would feel it for sure. But apologies, my mistake. It isn't the cannonball doing 1000m/s. It's you. So where's the kinetic energy now? Can you feel it coursing through your veins? No. Because what’s really there is mass, and relative motion.

You can also feel heat. Touch that stove and you feel that heat. We talk about heat exchangers and heat flow as if there’s some magical mysterious fluid in there. And yet we know there isn’t. We know that heat is another derived effect of motion.

(https://www.thenakedscientists.com/forum/proxy.php?request=http%3A%2F%2Fwww.uwsp.edu%2Fphysastr%2Fkmenning%2Fimages%2Fmolecules.gif&hash=17e1b7a0e8b6259205098d811bc58e85)

Taste is chemical in nature, and somewhat primitive. Most of your sense of taste is in fact your sense of smell. Do you know how smell works? Look up olfaction and you’ll learn about molecular shape. But the latest theory from a man called Luca Turin says it’s all down to molecular vibration, because isomers smell the same. That’s motion again.

The point of all this is there’s a lot of motion out there, and most of your senses are motion detectors. But it probably never occurred to you because you’re accustomed to thinking about the world in terms of how you experience it, rather than the scientific, empirical, fundamental, ontological things that are there.

And nowhere is this more so than with time.

So, what is time? Let’s start by looking up the definition of a second:

Under the International System of Units, the second is currently defined as the duration of 9,192,631,770 periods of the radiation corresponding to the transition between the two hyperfine levels of the ground state of the caesium-133 atom...

So, a second is nine billion periods of radiation. But what is a period? We know that radiation is electromagnetic in nature, the thing we commonly call light. We also know that light has a frequency. So let’s look at frequency:

Frequency = 1 / T and Frequency = v / λ

This says frequency is the reciprocal of the period T, and is also velocity v divided by wavelength λ . Combining the two, we can say T = λ / v, which means a period T is a wavelength λ divided by a velocity v. To try to find out more, we can drill down into wavelength and velocity. We know that a wavelength is a distance, a thing like a metre:

The metre is the length of the path travelled by light in vacuum during a time interval of 1/299792458 of a second...

And we know already that a velocity is a distance divided by a time. So if a period is a wavelength divided by a velocity, that means a period is a distance divided by a distance divided by a time. So let’s do some simple mathematics. Let’s work it through. We can combine T = λ / v and v = λ / t and write it down as:

T = λ / ( λ / t)

Then we can cancel out the λs to get:

T = 1/(1/t)

Then we cancel the double reciprocal to leave:

T = t

The answer we get is T = t. A period of time is a period of time. This mathematical definition of time is circular. The mathematics tells us nothing about its base terms. So what is its true nature? How do we dig down and get to the bottom of it?

Let’s look at frequency some more. What’s the definition in English?

Frequency is the measurement of the number of times that a repeated event occurs per unit of time.

Our unit of time is the second. Frequency is the number of events per second. A second is nine billion periods of electromagnetic radiation. A period of radiation is an electromagnetic event, caused by an electromagnetic event happening inside an atom. For an event to happen, something has to move. Some component of the caesium atom has to travel some distance. A hyperfine transition is to do with magnetic dipole movement, a flip-flop interaction between the nucleus and an electron. It’s magnetic, so it’s electromagnetic in nature. Like the electron is electromagnetic in nature. Like the photon is electromagnetic in nature. So in some simple respect, we can consider some vital component of the atom to be electromagnetic just like light.
 
(https://www.thenakedscientists.com/forum/proxy.php?request=http%3A%2F%2Fwww.astro.rug.nl%2F%7Eweygaert%2FInleidingStk2%2Fimages%2Fspinflip.FG18_016.jpg&hash=914052ffc97e941de439f33e0147e8f9)

The answer comes with a rush. It’s a form of light moving inside the atom, and it causes more light, radiation with a frequency, waves with peaks, We sit there counting them as they go by, and when we get to nine billion, we say its a second. Then we use this second to measure the speed of light. We measure the speed of light in terms of the speed of light. In caesium atoms, in hydrogen atoms, in our own atoms, in the atoms of everything. No wonder it never changes.

And so the penny drops: the interval between events is measured in terms of other events. And the interval between those events is measured in terms of other events. Until there are no events left, only intervals. And intervals are frozen timeless moments. But you need events, not frozen timeless intervals to mark out the time. The events aren’t in the time, the time is in the events. Because time is merely the measure of events, of change, measured against some other change. And for things to change, there has to be motion. You don’t need time to have motion. You need motion to have time.

You don’t need regular atomic motion to mark out time. Any regular motion will do. Yes, we counted nine billion oscillations and called it second. One, two, three… nine billion. But you don’t have to count microwave wavepeaks emitted by a caesium atom. You could count beans in a bucket. Ping, ping, ping, chuck them in, regular as clockwork.

(https://www.thenakedscientists.com/forum/proxy.php?request=http%3A%2F%2Fhighcountrycoffee.com%2Fecommerce%2Fos%2Fcatalog%2Fimages%2Fbucket-of-beans.gif&hash=af1f62025fac9caa994a678704a4f83e)

You’re sitting there counting beans into the bucket, ping, ping, ping, regular as clockwork. Now, what is the direction of time? The only direction that is actually there, is the direction of the beans you’re throwing, and that direction is the direction of motion through space. A fuller bucket is not the direction of time. More beans is not the direction of time. The direction of time is the direction of your counting, and I could have asked you to count the beans out of the bucket. There is no real direction. It’s as imaginary as the direction you take when you count along the set of integers.

1 2 3 4 5 6 7 8 9 10 11 12 13 14 15 16 17 18 19 20 21 22 23 24 25 26 27 28 29 →

It’s imaginary, so you cannot actually point in this direction. Nor can an arrow. There is no Arrow of Beans, so there is no Arrow of Time. And since there’s no direction, there’s no direction you can possibly travel in. And since you can’t travel, you can’t travel a length, and a length can’t pass you by. It’s all abstraction, a false concept rooted in the language we use to think. Yet we never ever think about what the words actually mean. Instead we say the clock is running slow as if a clock is an athlete. We say the day went quickly but it didn’t go anywhere. We say years pass, but they don’t go by like buses.

(https://www.thenakedscientists.com/forum/proxy.php?request=http%3A%2F%2Ft0.gstatic.com%2Fimages%3Fq%3Dtbn%3AKYaAzWCvb76nWM%3Ahttp%3A%2F%2Fallstatepropertyonline.com%2Ffiles%2FRunningClock.gif&hash=7b5c9742c335008b863142d84b9a9e1f)

The only directions that are there, are the directions of the spatial motions that make the events that we use to measure the intervals between the other events. What’s there is the motion of light, the motion of atoms, and the motion of clocks, buses, and rivers. What’s there is the motion of the earth, and the sun, moon, and stars. And these motions are being counted, incremented, added up. We count regular atomic motion to use as a ratio against some other motion, be it of light, clocks, or buses. All of these things have motion, both internal motion and travelling motion. And all those motions are real, with real directions in space. But the time direction isn't real. It's as imaginary as a trip to nine billion.

That's why the past is only in your head, in your memory, in your records. It isn’t a place you can travel to. It’s just the places where things were. All those places that are still here in the universe. And while the past is the sum of all nows, now lasts for no time at all. Because there’s no time like the present, and time needs events, and when you take away the events, you take away the time. A second isn’t some slice of spacetime. It’s just nine billion motions of light from a caesium atom. Accelerate to half the speed of light and a second is still nine billion motions of light from a caesium atom. But there's only half the local motion there used to be, because the other half is already doing the travelling motion through space. That’s why time dilates.

It’s easy to understand time dilation. Imagine yourself as a metronome. Each tick is a thought in your head, a beat in your heart, a second of your time. If you’re motionless with respect to me I see you ticking like this: |||. If you flash by in a spaceship, I see you ticking like this: /\/\/\. If you could reach c and we know you can’t, you wouldn’t tick at all. Your time would flatline like this ______ because any transverse motion would cause c to be exceeded. You wouldn’t tick for me, you wouldn’t tick for you, and you wouldn’t tick for anybody else in the universe.

That’s the thing we’re interested in. The universe. That’s the thing that’s out there, the thing we’re a part of, the thing we’re trying to understand. It’s full of motion, and this is what it’s like:

(https://www.thenakedscientists.com/forum/proxy.php?request=http%3A%2F%2Fupload.wikimedia.org%2Fwikipedia%2Fcommons%2F6%2F6d%2FTranslational_motion.gif&hash=0657f9abf0cb8d4e595a08cda44fe9b8)

What can you see? What can you measure? You can measure the height. You can measure the width. And if it wasn't just a picture you could measure the depth. That's three Dimensions, with a capital D because we have freedom of movement in those dimensions. What else can you see? What else can you measure? You might imagine a fourth dimension, a time dimension. But the picture comes from the wikipedia temperature page. It’s a gif, a moving image, and in that image, those red and blue dots are moving. The thing you can measure is temperature.

Temperature is an aspect of heat, an emergent property, a derived effect of atomic and molecular motion. When you measure the temperature, you are measuring an aggregate motion. If you were one of those dots, you would not talk of climbing to a “higher temperature”. There is no real height. You can’t literally climb to a higher temperature. Hence we don’t call temperature a dimension. But people did. Temperature used to be called a dimension, but the word has gradually changed from its original meaning of “measure”, and is now assumed to be something that offers a degree of freedom, something you can move through.

We are immersed in time like the dots are immersed in temperature. It’s a different measure, but just as we cannot travel in temperature because there is no real height, we cannot travel in time because there is no real length. Because time is a dimension with a small d. There is no degree of freedom. I can hop backwards a metre but not backwards a second. Because time is a measure of change rather than a measure of place, and it has no absolute units, because you can only measure one change of place against another. It’s a relative measure of motion. The units are relative, and that’s what Special Relativity was telling us all along.

Special Relativity tells us that your relative velocity alters your measurement of space and time compared to everybody else. You increase your relative velocity and space appears to contract while time dilates by a factor of √(1-v²/c²). If you travel at .99c, space appears to contract to one seventh of its former size. So your trip to a star seven light years away only takes you a year. But physics is about the universe, and in that universe it took you seven years. The star didn’t become a disc because you flashed by. The space in the universe didn’t really contract because you travelled through it. But your time did.

(https://www.thenakedscientists.com/forum/proxy.php?request=http%3A%2F%2Fwww.thebigview.com%2Fspacetime%2Ftdgraphformula.gif&hash=b39fbe077f4e339f2802fec1adc69067)

Einstein didn’t quite understand the full meaning of relativity until later in life. He started off by saying there is no absolute time, using a postulate that says the the speed of light is always measured to be the same. But when he did general relativity, he said the speed of light varies with position. It it wasn’t until he was with Godel in Princeton that he really got it:
 
"It is a widely known but insufficiently appreciated fact that Albert Einstein and Kurt Godel were best friends for the last decade and a half of Einstein's life. They walked home together from Princeton's Institute for Advanced Study every day; they shared ideas about physics, philosophy, politics, and the lost world of German-Austrian science in which they had grown up. What is not widely known is that in 1949 Godel made a remarkable discovery: there exist possible worlds described by the theory of relativity in which time, as we ordinarily understand it, does not exist. He added a philosophical argument that demonstrates, by Godel's lights, that as a consequence, time does not exist in our world either. If Godel is right, Einstein has not just explained time; he has explained it away... (Palle Yourgrau, A World Without Time)".

(https://www.thenakedscientists.com/forum/proxy.php?request=http%3A%2F%2Fedge.org%2Fimages%2FEinstein.Godel.550.jpg&hash=19b6943b4f2e8e357816cc418e011c7d)

And what he got was this: time exists like heat exists. It’s real because it does things to us. But just like heat it’s an emergent property, a derived effect of motion. It means time is not fundamental. It isn’t a dimension like the dimensions of space. We don’t see four dimensions. We see space and motion through it. The thing called c is a conversion factor, between the measure of distance and the measure we call time, and both are derived from the motion of light. It’s the motion that’s king, the universe is not a block universe, it is a world in motion. The worldlines are only in mathematical space, and in your head. There’s no place that’s the future, and no place that’s the past. There’s only this place, and the time is always now. We don’t travel in time at one second per second. We don't travel in time at all. To travel backwards in time we'd need to unevent events, we’d need negative motion. But motion is motion whichever way it goes. You can’t have negative motion, just as you can’t have negative distance. Just as you can’t have negative carpets. So you can’t travel in time. There are no time travel paradoxes, because there is no time travel, and there is no time travel because time is just a relative measure of motion. And motion is travel. You can’t travel through travel.

So those celebrity physicists who talk earnestly of time machines are wrong. Dead wrong. Not even wrong. And all those folk who puzzle about the beginning of time are chasing the wrong horse. There never was any beginning of time. Time didn’t start thirteen point seven billion years ago. Because time didn’t start in the first place. It was motion that started in the first place. It was a place, not a time. And it’s this place, the place we call the universe, marked out by every light path you can track through timeless space. That’s how far we’ve come. A long long way, in no time at all

Farsight "Time exist as heat exists" is a pure illusion of your mind. You take a hot stone in cold room. Stone will "coll" down. Not in time, in space only. And numerical order of stone colling down you measure with clocks: t0, t1, t2, t3.....tn. Lets say tn is 2500 second. in 2500 second stone has same temperature as room. Clock tick in space only, stone get coll in space only. yours amrit
Title: Will a photon clock run at a different rate from an atomic clock under gravity?
Post by: amrit on 29/05/2010 21:53:02
in proper english:
Farsight "Time exists as heat exists" is a pure illusion of your mind. You take a hot stone in cold room. Stone will "cool" down. Not in time, in space only. And numerical order of stone cooling down you measure with clocks: t0, t1, t2, t3.....tn. Lets say tn is 2500 second. in 2500 second stone has same temperature as room. Clock tick in space only, stone get cool in space only. yours amrit
Title: Will a photon clock run at a different rate from an atomic clock under gravity?
Post by: amrit on 30/05/2010 21:23:15
Farsight did not answer yet, we can turn back to the initial subject: invariance of light velocity in SR and GR implies that photon clock has no relativistic diminishing of velocity:
http://vixra.org/abs/1005.0073
Title: Will a photon clock run at a different rate from an atomic clock under gravity?
Post by: JP on 31/05/2010 05:21:46
Farsight,

Do you have any mathematics to back up your proposed theory?  It's impossible to place it in the context of observations and determine how it differs from the mainstream theory unless you give us something more than pictures and words...
Title: Will a photon clock run at a different rate from an atomic clock under gravity?
Post by: Geezer on 31/05/2010 07:48:15
To support your position that there is no relative speed difference (and therefore time as we know it does not exist) I think you will have to produce an alternative definition for whatever it is you maintain is the thing we currently know as time.
There must be some misunderstanding here. I've said there is a relative speed difference, challenging amrit's OP assertion. I've also said time exists like heat exists. It's just that it isn't something that really flows, and we don't really travel through it. 

Farsight, this is I was referring to. It could be a bit ambiguous.

"You might have some difficulty accepting this, but there's absolutely no scientific evidence for time is running slower. None whatsoever."
Title: Will a photon clock run at a different rate from an atomic clock under gravity?
Post by: amrit on 31/05/2010 09:12:57
To support your position that there is no relative speed difference (and therefore time as we know it does not exist) I think you will have to produce an alternative definition for whatever it is you maintain is the thing we currently know as time.
There must be some misunderstanding here. I've said there is a relative speed difference, challenging amrit's OP assertion. I've also said time exists like heat exists. It's just that it isn't something that really flows, and we don't really travel through it. 

Farsight, this is I was referring to. It could be a bit ambiguous.

"You might have some difficulty accepting this, but there's absolutely no scientific evidence for time is running slower. None whatsoever."


Geezer, time runs in the mind. Time “past-present-future” (psychological time) is a mind construct through which we experience motion into timeless universe.
Through psychological time we experience timelessness of the universe as “present moment”.
Once we are aware of psychological time we experience timelessness of the universe as “eternal present moment” that has its place in space. In the space is always NOW.
Title: Will a photon clock run at a different rate from an atomic clock under gravity?
Post by: graham.d on 31/05/2010 10:04:30
Amrit, I find your repetition of your concept of time to be unhelpful in giving me (and maybe others) an understanding of your ideas. There is nothing in what you say that seems to relate to the physics. In stating the universe is "Timeless" you may be trying to get people to think in another way, but this still does not relate to the physics. And the experiment showing difference in behaviour between a "photon clock" and other clocks is (IMHO) not going to be successful. However I don't see how this confirms or denies your beliefs in any case. 
Title: Will a photon clock run at a different rate from an atomic clock under gravity?
Post by: amrit on 31/05/2010 13:15:24
Amrit, I find your repetition of your concept of time to be unhelpful in giving me (and maybe others) an understanding of your ideas. There is nothing in what you say that seems to relate to the physics. In stating the universe is "Timeless" you may be trying to get people to think in another way, but this still does not relate to the physics. And the experiment showing difference in behaviour between a "photon clock" and other clocks is (IMHO) not going to be successful. However I don't see how this confirms or denies your beliefs in any case. 

Graham there is no single evidence that time exist as a physical reality. Our concept of BLOCK UNIVERSE resolves this puzzle in details: http://vixra.org/abs/1005.0098

Tell me one single experiment in physics that proves existence of time!

FQXI gives 2 million dollars grant for basic research on time.
http://www.fqxi.org/grants/large/initial
If with time all will be clear you think they would give such big money for research?

Title: Will a photon clock run at a different rate from an atomic clock under gravity?
Post by: graham.d on 31/05/2010 14:16:10
Amrit, I am not denying that the concepts have some merit but just that you are not explaining them well, at least to me. I have read through the paper but have difficulty following your reasoning and I think to do so would involve following up many references.  Statements like "Tell me one single experiment in physics that proves existence of time!" are not convincing because, philosophically, nothing is provable absolutely but only if based on specific premises. You tell me one single experiment in physics that proves the existence of space, for example.
Title: Will a photon clock run at a different rate from an atomic clock under gravity?
Post by: Farsight on 31/05/2010 17:44:04
Farsight, Do you have any mathematics to back up your proposed theory?  It's impossible to place it in the context of observations and determine how it differs from the mainstream theory unless you give us something more than pictures and words...
The mathematics isn't any different to what you know, JP. It's a difference in interpretation, in what the mathematics means. And it's backed by the observational evidence in that we do see things moving, but we don't see time flowing. This isn't my theory by the way. This goes back to Aristotle. Also take a look at Presentism.   
Title: Will a photon clock run at a different rate from an atomic clock under gravity?
Post by: Farsight on 31/05/2010 17:54:31
Farsight, this is I was referring to. It could be a bit ambiguous.

"You might have some difficulty accepting this, but there's absolutely no scientific evidence for time is running slower. None whatsoever."
Yes, it is ambiguous. It would have been clearer if I'd said:

"There's no actual scientific evidence to support the concept of time "running". That's just a figure of speech. It isn't what you see, what you see is things moving. When the motion is going slower, we say time is running slower, but again, it's just a figure of speech associated with the way we usually think about time."
Title: Will a photon clock run at a different rate from an atomic clock under gravity?
Post by: Farsight on 31/05/2010 18:05:17
Graham: re experiment proving the existence of space, simply hold your hands up a metre apart. There's a gap between them. You can see that gap. That's space. You can't see anything there, but the gap is there, and you can see that it's there. Now waggle your hands and you can see motion. Hence you can demonstrate the existence of space and motion quite easily. However you can't similarly demonstrate time running or flowing, or motion through time.
Title: Will a photon clock run at a different rate from an atomic clock under gravity?
Post by: amrit on 31/05/2010 18:14:25
Amrit, I am not denying that the concepts have some merit but just that you are not explaining them well, at least to me. I have read through the paper but have difficulty following your reasoning and I think to do so would involve following up many references.  Statements like "Tell me one single experiment in physics that proves existence of time!" are not convincing because, philosophically, nothing is provable absolutely but only if based on specific premises. You tell me one single experiment in physics that proves the existence of space, for example.

yes we can observe only distances between objects and not space itself, for "space" we understand medium in which objects exists and this medium is timeless
Title: Will a photon clock run at a different rate from an atomic clock under gravity?
Post by: amrit on 31/05/2010 18:17:51
Farsight, Do you have any mathematics to back up your proposed theory?  It's impossible to place it in the context of observations and determine how it differs from the mainstream theory unless you give us something more than pictures and words...
The mathematics isn't any different to what you know, JP. It's a difference in interpretation, in what the mathematics means. And it's backed by the observational evidence in that we do see things moving, but we don't see time flowing. This isn't my theory by the way. This goes back to Aristotle. Also take a look at Presentism.  

Farsight im waiting you comment on hot stone that cool down in cold room.....you ca not just ignoring that.....be more cool

PS what you say:

Graham: re experiment proving the existence of space, simply hold your hands up a metre apart. There's a gap between them. You can see that gap. That's space.

is a pure disaster
Title: Will a photon clock run at a different rate from an atomic clock under gravity?
Post by: graham.d on 31/05/2010 18:52:35
Graham: re experiment proving the existence of space, simply hold your hands up a metre apart. There's a gap between them. You can see that gap. That's space. You can't see anything there, but the gap is there, and you can see that it's there. Now waggle your hands and you can see motion. Hence you can demonstrate the existence of space and motion quite easily. However you can't similarly demonstrate time running or flowing, or motion through time.

My point was not to have a simple visual demonstration of space. What you are showing is that I can define a distance between my outstretched hands and compare it with, say, a metre rule. Of course this is providing the rule is not moving relative to me when it gets more involved and takes the concept to a different level from observation. On the same basis I can compare the rate of my breathing with a clock ticking seconds. That is equally convincing isn't it? I was questioning the concept of "proof" and how this would differ between its application to time compared with space.
Title: Will a photon clock run at a different rate from an atomic clock under gravity?
Post by: Geezer on 31/05/2010 20:46:31
Farsight, this is I was referring to. It could be a bit ambiguous.

"You might have some difficulty accepting this, but there's absolutely no scientific evidence for time is running slower. None whatsoever."
Yes, it is ambiguous. It would have been clearer if I'd said:

"There's no actual scientific evidence to support the concept of time "running". That's just a figure of speech. It isn't what you see, what you see is things moving. When the motion is going slower, we say time is running slower, but again, it's just a figure of speech associated with the way we usually think about time."

Well, you can't really say "the motion is going slower" either. "Slower" suggests comparative speed, but speed itself is a function of time, so differences in local time have no effect on speed.

That's why I think it's necessary to create new terminology because so many concepts have an obvious or less obvious function of time built into them.
Title: Will a photon clock run at a different rate from an atomic clock under gravity?
Post by: Farsight on 01/06/2010 13:56:57
Amrit, I responded to your PM re your question. I'll see if I can find what I said and post it up.

Edit: I can't see my sent messages. I said "time exists like heat exists" because heat is an emergent property of motion. A gas molecule doesn't have any fundamental property of heat, it has a velocity and a kinetic energy. However a container full of fast-moving molecules is said to be "hot" because temperature is a measure of the average motion of the gas molecules. And heat burns, it exists all right, even if it doesn't exist at the fundamental level. Time is a similar emergent property, but it's a cumulative measure of motion rather than an average measure. Your hot stone cools down because its atoms/molecules move slower, and we measure the rate of the slowdown against a clock, which "clocks up" some other regular motion as a benchmark. When you stop the clock, you stop motion, not time. If you stop all the motion in the universe, including the motion of light and in the body and brain of the observer, you can't "measure time" any more. That's because you never were really measuring time, you were measuring motion.   

My point was not to have a simple visual demonstration of space. What you are showing is that I can define a distance between my outstretched hands and compare it with, say, a metre rule. Of course this is providing the rule is not moving relative to me when it gets more involved and takes the concept to a different level from observation. On the same basis I can compare the rate of my breathing with a clock ticking seconds. That is equally convincing isn't it?
No. It doesn't offer any convincing evidence of "time flowing". You're just comparing two moving things. Substitute your beathing for another clock to appreciate this, and remember that a clock is "clocking up" motion. It displays some sort of cumulative counter of how many times a cog has gone round or a crystal has vibrated.

Well, you can't really say "the motion is going slower" either. "Slower" suggests comparative speed, but speed itself is a function of time, so differences in local time have no effect on speed.
That's a circular argument, Geezer. If you compare two clocks, then if they don't "keep time" the cogs in one are moving slower than the cogs in another. It doesn't matter if one is on the surface of the earth whilst the other is up in space, that's what's actually happening. You can see this. You can't see "time running slower". You can't see time "running" at all. One doesn't need a new terminology for this, just an adherence to the observational evidence and an appreciation that some of the things we say are figures of speech, because time is a function of motion, not the other way around. 
Title: Will a photon clock run at a different rate from an atomic clock under gravity?
Post by: imatfaal on 01/06/2010 16:13:59
My point was not to have a simple visual demonstration of space. What you are showing is that I can define a distance between my outstretched hands and compare it with, say, a metre rule. Of course this is providing the rule is not moving relative to me when it gets more involved and takes the concept to a different level from observation. On the same basis I can compare the rate of my breathing with a clock ticking seconds. That is equally convincing isn't it? I was questioning the concept of "proof" and how this would differ between its application to time compared with space.

Couldn't agree more Graham. I think Amrit and Farsight are not accepting that the received conception of time is axiomatic to current physics, as is our conception of space.  As axioms, these concepts are fundamental and beyond proof - the first sections of Einstein's easy guide to SR and GR explains this concept of axioms beautifully; I am sure I have seen a link to the text recently and will post when I find it. 

got it http://en.wikisource.org/wiki/Relativity:_The_Special_and_General_Theory/Part_I

On a philosophical level this question becomes more interesting - but less useful.  As this is being advanced as a scientific theory could someone propose a real world experiment that would give a result that would vary from that expected under current dogma.  I think that experiments have already been performed that contradict the original idea - but the work required to demonstrate this is too much for me at present.  Matthew

Title: Will a photon clock run at a different rate from an atomic clock under gravity?
Post by: Geezer on 01/06/2010 17:58:18
Well, you can't really say "the motion is going slower" either. "Slower" suggests comparative speed, but speed itself is a function of time, so differences in local time have no effect on speed.
That's a circular argument, Geezer. If you compare two clocks, then if they don't "keep time" the cogs in one are moving slower than the cogs in another. It doesn't matter if one is on the surface of the earth whilst the other is up in space, that's what's actually happening. You can see this. You can't see "time running slower". You can't see time "running" at all. One doesn't need a new terminology for this, just an adherence to the observational evidence and an appreciation that some of the things we say are figures of speech, because time is a function of motion, not the other way around. 

Farsight: We use motion, chemical reactions, sub-atomic activity, etc., etc. to observe time, but that is hardly evidence that time only exists because of motion. Motion can only be determined on the basis of time. Without time, motion boils down to "things can be in different places".

As I said, many concepts have a function of time built into them, and motion is one of them. If you want to establish an alternative definition for time, you'll have to define it in terms that do not include a function of time, otherwise your definition will be recursive.

EDIT:

Come to think of it, if, as you say, time is a function of motion, then motion, by definition, is a function of time. I suppose you could define everything in terms of motion, but I don't think there is much point unless you can find a way to eliminate time from the equations. However, as motion is a function of time (and vice versa) that may not be possible.
Title: Will a photon clock run at a different rate from an atomic clock under gravity?
Post by: Farsight on 04/06/2010 15:54:25
Couldn't agree more Graham. I think Amrit and Farsight are not accepting that the received conception of time is axiomatic to current physics, as is our conception of space. As axioms, these concepts are fundamental and beyond proof - the first sections of Einstein's easy guide to SR and GR explains this concept of axioms beautifully; I am sure I have seen a link to the text recently and will post when I find it. Got it http://en.wikisource.org/wiki/Relativity:_The_Special_and_General_Theory/Part_I
I have no issue with space, Matthew. But that received conception of time is to be blunt, wrong. It's unsupported by scientific evidence. There is no proof whatsoever that we "travel in time" or that "time flows". Have a look at "A World without Time: The Forgotten Legacy of Godel and Einstein" (http://www.amazon.co.uk/World-Without-Time-Forgotten-Einstein/dp/0713993871) re the view Einstein adopted in 1949. I like to think that I'm fully in line with this view, but do note that the front flap is a little misleading. Time as we ordinarily understand it does not exist. That's not to say that time doesn't exist. It just isn't what people think it is. 

On a philosophical level this question becomes more interesting - but less useful. As this is being advanced as a scientific theory could someone propose a real world experiment that would give a result that would vary from that expected under current dogma. I think that experiments have already been performed that contradict the original idea - but the work required to demonstrate this is too much for me at present.  Matthew
IMHO it's very useful indeed. But there are no different results to be had. One merely sees existing results in a new light, and then gains understanding that was previously lacking.


Farsight: We use motion, chemical reactions, sub-atomic activity, etc., etc. to observe time, but that is hardly evidence that time only exists because of motion.
You
missed the crucial point, Geezer. We observe motion, but we don't actually observe time.

Motion can only be determined on the basis of time. Without time, motion boils down to "things can be in different places".
You're still not recognising what we actually see. Time can only be determined on the basis of motion. Without motion, there isn't any time.

As I said, many concepts have a function of time built into them, and motion is one of them. If you want to establish an alternative definition for time, you'll have to define it in terms that do not include a function of time, otherwise your definition will be recursive.
One defines it using the motion we actually observe. Then one retains a function called time, but one now realises that we can't move through it and it doesn't actually flow.

Come to think of it, if, as you say, time is a function of motion, then motion, by definition, is a function of time.
There's no justification for that. 

I suppose you could define everything in terms of motion, but I don't think there is much point unless you can find a way to eliminate time from the equations. However, as motion is a function of time (and vice versa) that may not be possible.
No, you can't remove time from the equations. You wouldn't want to anyway. But when you appreciate that the thing we call time is measured and defined using motion through space, you get a better concept of say gravity and electromagnetism, and you don't get distracted by science fiction such as time travel.
Title: Will a photon clock run at a different rate from an atomic clock under gravity?
Post by: Geezer on 04/06/2010 18:19:39
Come to think of it, if, as you say, time is a function of motion, then motion, by definition, is a function of time.
There's no justification for that. 

If A is a function of B, B is automatically a function of A. The justification is inescapable.
Title: Will a photon clock run at a different rate from an atomic clock under gravity?
Post by: JP on 05/06/2010 04:26:28
Couldn't agree more Graham. I think Amrit and Farsight are not accepting that the received conception of time is axiomatic to current physics, as is our conception of space. As axioms, these concepts are fundamental and beyond proof - the first sections of Einstein's easy guide to SR and GR explains this concept of axioms beautifully; I am sure I have seen a link to the text recently and will post when I find it. Got it http://en.wikisource.org/wiki/Relativity:_The_Special_and_General_Theory/Part_I
I have no issue with space, Matthew. But that received conception of time is to be blunt, wrong. It's unsupported by scientific evidence. There is no proof whatsoever that we "travel in time" or that "time flows". Have a look at "A World without Time: The Forgotten Legacy of Godel and Einstein" (http://www.amazon.co.uk/World-Without-Time-Forgotten-Einstein/dp/0713993871) re the view Einstein adopted in 1949. I like to think that I'm fully in line with this view, but do note that the front flap is a little misleading. Time as we ordinarily understand it does not exist. That's not to say that time doesn't exist. It just isn't what people think it is.

Before someone goes out and spends money on a book that's been poorly reviewed, I would suggest they read up on it.  This seems like a pretty good review: http://www.ams.org/notices/200707/tx070700861p.pdf

Also, you certainly have very little evidence for telling people that the mainstream view of time is "wrong."  What you're claiming is philosophy with no mathematics to back it up.  Science is about making predictions and observations--the validity of a theory is based on how well it seems to model reality and how well it matches experiments.  Your philosophy doesn't offer any predictions, so it isn't science.  It's not even clear that it's consistent with the mainstream view of time as a dimension space-time.  Therefore, claiming that your philosophy is a scientific fact supported by evidence is misleading.
Title: Will a photon clock run at a different rate from an atomic clock under gravity?
Post by: Farsight on 05/06/2010 16:11:02
Before someone goes out and spends money on a book that's been poorly reviewed, I would suggest they read up on it. This seems like a pretty good review: http://www.ams.org/notices/200707/tx070700861p.pdf
I'm afraid it isn't. Stachel muddies the water and evades the essential point in order to defend an unsupported position via circular argument.

Also, you certainly have very little evidence for telling people that the mainstream view of time is "wrong."
I've given ample evidence in Time Explained (http://www.thenakedscientists.com/forum/index.php?topic=31723.msg311238#msg311238). If you disagree, try to show where the evidence I offer is incorrect.   

What you're claiming is philosophy with no mathematics to back it up. Science is about making predictions and observations...
As I've said, the mathematics is unchanged. And it certainly isn't philosophy, it's phenomenology. Because science is about observations, and there are no observations whatsoever to support the idea that time flows or that we travel through it. If you beg to differ, I challenge you to offer some. 

The validity of a theory is based on how well it seems to model reality and how well it matches experiments. Your philosophy doesn't offer any predictions, so it isn't science.  It's not even clear that it's consistent with the mainstream view of time as a dimension of space-time. Therefore, claiming that your philosophy is a scientific fact supported by evidence is misleading.
No it isn't misleading, and again it is not philosophy. The observational evidence tells us what's scientific fact, and I adhere to it whilst adhering to special relativity. See page 31 of The Meaning of Relativity where Einstein says "The non-divisibility of the four-dimensional continuum of events does not at all, however, involve the equivalence of the space co-ordinates with the time co-ordinate". I'm giving you the science. The flow of time and travelling through time is the philosophy that leads to the reductio-ad-absurdum of the grandfather paradox.

Time travel isn't mainstream, JP. It's science fiction. The "stasis box" is science-fiction too, but it's useful to point out the obvious: get in the box, and the "stasis field" prevents all motion, even at the atomic level. So you can't move, your heart doesn't beat, and you can't even think. When I open the box five hundred years later, to you it's like I opened the box as soon as you got in. You "travelled" to the future by not moving at all. Instead everything else did. And all that motion, be it the motion of planets or people or atoms or light, was through space.
Title: Will a photon clock run at a different rate from an atomic clock under gravity?
Post by: Farsight on 05/06/2010 16:21:25
If A is a function of B, B is automatically a function of A. The justification is inescapable.
Not so. The temperature of a gas is a function of molecular motion involving an average. The opposite is not true, because a single molecule has a velocity, not a temperature. Your assertion puts cause on an equal footing with effect, and places emergent properties on a par with fundamental properties. It doesn't hold.
Title: Will a photon clock run at a different rate from an atomic clock under gravity?
Post by: PhysBang on 05/06/2010 16:59:48
If you disagree, try to show where the evidence I offer is incorrect. 
Over the breadth of the internet there is a veritable book about how wrong your argument is.

The funniest is that the same "mathematics" that you use to show that time is circularly defined shows that space is circularly defined and shows that every physical quantity is circularly defined. But philosophers of science have known this for decades if not centuries.

The best argument against your argument is that you ignore your own circularity as pointed out by your favourite source for quotation  mining: Einstein. As Einstein points out in 1905, we cannot provide a measurement of space without providing some definition of what it means to be "the same time" as different points that are separated by a distance. Only once we do that can we have a real physical definition of measured distance and only then can we have a definition of motion and at rest. But Einstein also points out that we are perfectly free to chose from an infinite number of ways of setting up, physically, what is "the same time" at different distances. This means that there are an infinite number of measurements of space and thus an infinite number of ways of defining any given motion (or even whether or not there is motion). But we cannot have a physical idea of motion without some previous definition of space and time. This is just the conceptual facts.

Just because what we metaphorically call "movement through time" and "movement through spacetime" is different from what we call movement through space does not obviate us from actually understanding what these phrases mean and it does not make the science and philosophy behind the real meaning of the phrases incorrect.

One incorrect idea about relativity that has its origin in the popular press is that relativity tells us that the universe changes when we are in motion. This is incorrect. In different circumstances, certain descriptions of the universe might be easier to describe or be easier for us to describe using certain measurement devices, but these descriptions are correct regardless of the motion or not of any given observer. To claim that the universe changes because of an observer is incorrect. It is just as incorrect to say that anything changes for a given observer or particle because they are in motion. Time goes on for any given particle just as it always does, what changes is the relationship between events as timed out for one description and events as timed out for another description. Without accepting this, the mathematics of relativity theory simply does not work.

Finally, what Einstein believed or did not believe about relativity theory is irrelevant. What matters is the actual science as handed down to the scientific community and as tested over and over again by careful study. That theory is not a theory without time, it is a theory with a very special relationship between space and time. To trust one's knowledge of this theory to comeone unwilling to actually learn or discuss the mathematics is foolish.
Title: Will a photon clock run at a different rate from an atomic clock under gravity?
Post by: Geezer on 05/06/2010 19:48:38
If A is a function of B, B is automatically a function of A. The justification is inescapable.
Not so. The temperature of a gas is a function of molecular motion involving an average. The opposite is not true, because a single molecule has a velocity, not a temperature. Your assertion puts cause on an equal footing with effect, and places emergent properties on a par with fundamental properties. It doesn't hold.

Farsight:

Apparently my opinion on the subject of functions is a commonly held belief. As Wikipedia is not necessarily the most reliable source, feel free to identify any errors in the following, or did you mean to say something other than "function" when you said that time was a function of motion?

http://en.wikipedia.org/wiki/Function_(mathematics)

http://en.wikipedia.org/wiki/Inverse_function

Perhaps you can also reduce your statement regarding gas molecules to a mathematical relationship so that we can test its validity?
Title: Will a photon clock run at a different rate from an atomic clock under gravity?
Post by: JP on 06/06/2010 03:48:04
Also, you certainly have very little evidence for telling people that the mainstream view of time is "wrong."
I've given ample evidence in Time Explained (http://www.thenakedscientists.com/forum/index.php?topic=31723.msg311238#msg311238). If you disagree, try to show where the evidence I offer is incorrect.   

When you have to defend your theory with "show me where it's wrong," it's not a theory.  There's a reason that scientific theories are required to make mathematical predictions that are then verified by experiment.  You still have no mathematics and no physical theory.  You're giving us quotes from a variety of sources with no mathematics to back them up and asking us to overturn a successful theory that has plenty of mathematics and nearly a century of successful quantitative predictions. 

Finally, what Einstein believed or did not believe about relativity theory is irrelevant. What matters is the actual science as handed down to the scientific community and as tested over and over again by careful study. That theory is not a theory without time, it is a theory with a very special relationship between space and time. To trust one's knowledge of this theory to comeone unwilling to actually learn or discuss the mathematics is foolish.
I agree with PhysBang 100% on this.  Einstein also disagreed with quantum mechanics, and yet it would be absurd to claim quantum mechanics was wrong because Einstein once said so.  Quantitative predictions backed up by experiments are needed, not quotes.
Title: Will a photon clock run at a different rate from an atomic clock under gravity?
Post by: Farsight on 06/06/2010 09:49:32
Geezer: you brought up "function", not me. I only used the word in response to your usage. I said time is an emergent property of motion like heat is an emergent property of motion, time being a cumulative measure whilst temperature is an average.

When you have to defend your theory with "show me where it's wrong," it's not a theory. There's a reason that scientific theories are required to make mathematical predictions that are then verified by experiment. You still have no mathematics and no physical theory. You're giving us quotes from a variety of sources with no mathematics to back them up and asking us to overturn a successful theory that has plenty of mathematics and nearly a century of successful quantitative predictions.
I'm certainly not asking you to overturn relativity. I've already said that the mathematics is unchanged. What I'm asking you to do is look at the scientific evidence and appreciate that there is no evidence for travelling through time. The mathematics doesn't support it either, because we plot lines in Minkowski spacetime rather than moving through it.     

Finally, what Einstein believed or did not believe about relativity theory is irrelevant. What matters is the actual science as handed down to the scientific community and as tested over and over again by careful study. That theory is not a theory without time, it is a theory with a very special relationship between space and time. To trust one's knowledge of this theory to comeone unwilling to actually learn or discuss the mathematics is foolish.
I agree with PhysBang 100% on this. Einstein also disagreed with quantum mechanics, and yet it would be absurd to claim quantum mechanics was wrong because Einstein once said so. Quantitative predictions backed up by experiments are needed, not quotes.
What Einstein believed about relativity is most certainly relevant! And I'm  not doing away with time, I'm saying it exists like heat exists, but isn't something you travel through. Observations and scientific experiments back ME up, not time machines. Note that Einstein didn't disagree with quantum mechanics, he was in on the ground floor in 1905. That's what he got his Nobel Prize for. What he disagreed with was the Copenhagen Interpretation, the "meaning" of the mathematics. 
Title: Will a photon clock run at a different rate from an atomic clock under gravity?
Post by: Geezer on 06/06/2010 16:34:35
Geezer: you brought up "function", not me.

Farsight: Then why did you say "time is a function of motion, not the other way around" here? Was that just a "figure of speech" too?

One doesn't need a new terminology for this, just an adherence to the observational evidence and an appreciation that some of the things we say are figures of speech, because time is a function of motion, not the other way around. 
Title: Will a photon clock run at a different rate from an atomic clock under gravity?
Post by: Geezer on 07/06/2010 06:43:34
Looks like this is going nowhere. Thread locked.